Anda di halaman 1dari 68

Which of the following cells play a crucial role in the pathogenesis of alveolar-capillary damage in adult

respiratory distress syndrome (ARDS)?


A. D!-positive lymphocytes
". D#-positive lymphocytes
. $osinophils
D. %ast cells
$. &eutrophils
$'planation(
)he correct answer is $. ARDS* pathologically referred to as diffuse alveolar damage* is a clinical syndrome of
acute respiratory failure resulting from diffuse in+ury to the alveolar,capillary -arrier. Such in+ury may -e caused
-y a great variety of initiating insults* the most fre.uent of which are shoc/* severe trauma* sepsis* and gastric
aspiration. All these different forms of in+ury result in recruitment of neutrophils within the alveolar capillaries.
&eutrophils release chemo/ines that attract histiocytes and produce o'ygen radicals* prostaglandins* and
proteases that damage alveolar epithelium. 0ormation of hyaline mem-ranes is due to a com-ination of plasma
fluid e'travasation and alveolar cell necrosis.
D!1 (helper) lymphocytes (choice A)* D#1 (cytoto'ic) lymphocytes (choice ")* eosinophils (choice )* and
mast cells (choice D) have -een implicated in a num-er of pulmonary diseases* -ut not in diffuse alveolar
damage.
An animal is made dia-etic -y in+ection of a drug that destroys pancreatic 2-eta3 cells. Removal of which of the
following organs would most li/ely produce a decrease in -lood glucose concentration in this animal?
A. Anterior pituitary
". olon
. 4onads
D. 5idney
$. 6ancreas
$'planation(
)he correct answer is A. )wo of the secretions of the anterior pituitary affect the sensitivity of peripheral tissues
to the action of insulin. 4rowth hormone has a direct effect on liver and muscle to decrease insulin sensitivity.
)his may -e partly through a growth hormone-induced decline in insulin receptors or to un/nown post-receptor
defects. 7n e'cess* growth hormone is 8dia-etogenic*8 and a-out 9:; of patients with acromegaly have
dia-etes. A)< indirectly has anti-insulin effects -y virtue of the cortisol secretion it evo/es. =i/e growth
hormone* cortisol also decreases insulin sensitivity in peripheral tissues. A third anterior pituitary hormone*
)S<* also tends to increase -lood glucose levels. 7n this case* the effect is pro-a-ly mediated mostly through
increased glucose a-sorption -y the gut. 6atients with hyperthyroidism can sometimes e'hi-it a postprandial
glucosuria -ecause of e'cessive intestinal glucose a-sorption. 7n dia-etic animals* the removal of the anterior
pituitary may lower -lood glucose -y increasing tissue sensitivity to whatever insulin remains.
Removal of the colon (choice ") should have little effect on -lood glucose since dietary glucose is a-sor-ed in
the small intestine.
Se' steroids secreted -y the gonads (choice ) have little effect on -lood glucose concentration.
)he /idney (choice D) plays an important role in rea-sor-ing filtered glucose. 7n dia-etes* the tu-ular
rea-sorption ma'imum is e'ceeded and glucose spills over into the urine. )he loss of glucose in the urine helps
to reduce the severity of the plasma hyperglycemia. Removal of the /idneys would* if anything* ma/e the
hyperglycemia worse.
6ancreatectomy (choice $) would ma/e the hyperglycemia worse -y removing the source of any remaining
insulin.
A >>-year-old woman gives -irth to a -a-y girl. )he ne't day* she -egins to -leed from her vagina and from
venipuncture sites. =a-oratory studies demonstrate decreased platelets* prolonged prothrom-in time (6)) and
partial throm-oplastin time (6)))* and increased fi-rin split products. )hese features are most consistent with
which of the following?
A. Disseminated intravascular coagulation
". <emophilia A
. Severe liver disease
D. ?itamin 5 deficiency
$. ?on Wille-rand@s disease
$'planation(
)he correct answer is A. )he patient is e'periencing disseminated intravascular coagulation (D7)* a feared*
and often life-threatening complication of many other disorders* including amniotic fluid em-olism* infections
(particularly gram-negative sepsis)* malignancy* and ma+or trauma. )he diagnosis is suspected when -oth a
decrease in platelets and a prolongation of 6) and 6)) times are o-served. )he o-served hematologic
a-normalities are due to consumption of platelets and clotting factors* caused -y e'tensive microclot formation
with accompanying fi-rinolysis (reflected -y the increased fi-rin split products). )he D-dimer assay measures
cross-lin/ed fi-rin derivatives* and is a specific test for fi-rin degradation products.
<emophilia (choice ") will alter the 6)) without affecting the other indices.
Severe liver disease (choice ) produces alterations compara-le to those in vitamin 5 deficiency* and platelets
can also -e decreased secondary to a generaliAed meta-olic marrow dysfunction* -ut fi-rin split products would
not -e increased.
?itamin 5 deficiency (choice D) is associated with alterations in -oth 6) and 6))* -ut platelets will not -e
decreased* nor will fi-rin split products -e increased.
?on Wille-rand@s disease (choice $) produces impaired platelet adhesion and increases the -leeding time as
well as the 6))* -ut will not produce the other features descri-ed.
A :!-year-old woman presents with a pansystolic murmur along the lower left sternal -order radiating rightward to
the midclavicular line. )he murmur is medium pitched* has a -lowing .uality* and increases slightly on inspiration.
An S> is audi-le along the lower left sternal -order. Bugular venous pressure is elevated* and a prominent 8v8
wave is visi-le. Which of the following is the most li/ely etiology of the S>?
A. Aortic stenosis
". %itral regurgitation
. 6ulmonic stenosis
D. )ricuspid regurgitation
$. ?olume overloaded left ventricle
0. ?olume overloaded right ventricle
$'planation(
)he correct answer is 0. )he origin and radiation of the pansystolic murmur suggest tricuspid valve
incompetence. )his is further supported -y its pitch and .uality* and -y the fact that it increases on inspiration
when cardiac volume increases. )he regurgitant -lood flow from the ventricle during systole increases +ugular
venous -lood pressure and atrial v wave amplitude. )he origin of the S> sound* which occurs during early rapid
filling* is the filling of a volume-overloaded right ventricle. )he right ventricle overload is caused -y the
com-ination of systemic venous return and the return of the regurgitated -lood volume into the right ventricle.
Right ventricular failure and dilatation* with enlargement of the tricuspid valve orifice* is the most common cause
of tricuspid regurgitation and is often secondary to pulmonary hypertension or left ventricular failure.
Aortic stenosis (choice A) causes a harsh* shrill* midsystolic* crescendo-decrescendo murmur and would not
necessarily elevate right heart (and so +ugular venous) pressures. 7t is often associated with an S! (late rapid
filling) rather than an S>.
%itral regurgitation (choice ") causes a soft* -lowing* pansystolic murmur and is associated with an S>. 7t would
elevate left atrial pressures* not right atrial (and so +ugular venous) pressures.
6ulmonic stenosis (choice ) would also cause a crescendo-decrescendo murmur* not an S>.
)ricuspid regurgitation (choice D) is the source of the murmur -ut not the source of the S> sound.
A volume-overloaded left ventricle (choice $) could cause an S>* -ut tricuspid regurgitation will not cause a
volume overload in the left ventricle.
A routine physical e'amination demonstrates hypercalcemia in a !C-year-old man. irculatory levels of
parathyroid hormone are also elevated. $'ploratory surgery of the nec/ reveals diffuse hyperplasia of all four
parathyroid glands. Which of the following screening studies would most li/ely -e helpful in confirming the
diagnosis of multiple endocrine neoplasia* type 7 (%$& 7)?
A. 6entagastrin-simulated calcitonin secretion
". Serum epinephrine
. Serum gastrin
D. Serum norepinephrine
$. Drinary adrenaline(noradrenaline ratio
$'planation(
)he correct answer is . 7n multiple endocrine neoplasia* type 7 (%$& 7)* parathyroid hyperplasia and,or
adenomata are associated with pancreatic and duodenal endocrine tumors and endocrine hyperplasia. )he
most common secretory product of the pancreatic and duodenal endocrine lesions is gastrin. 6ituitary
adenomata may also occur in %$& 7.
6entagastrin-stimulated calcitonin secretion (choice A) is a mar/er for medullary carcinoma of the thyroid* which
is a component of %$& 77.
$pinephrine (adrenaline) is produced -y pheochromocytomas that occur as part of %$& 773 serum epinephrine
levels (choice ") and urinary adrenaline(noradrenaline ratios (choice $) may -e used for screening for
pheochromocytomas.
Serum norepinephrine (choice D) is usually not helpful in the diagnosis of pheochromocytoma* since serum
catecholamines can -e increased -y hypoglycemia* strenuous e'ertion* and central nervous system disease.
A !C-year old man complains of increasing difficulty in swallowing over the past > years. <e reports a feeling of
pressure in his chest occurring 9-> seconds after swallowing a solid -olus. <e also e'periences regurgitation of
undigested food eaten hours previously. A radiograph ta/en after swallowing -arium shows a distended
esophageal -ody with a smooth tapering at the lower esophageal sphincter. %anometry shows the a-sence of
esophageal peristalsis with swallowing and a lower esophageal sphincter that fails to rela'. What is the most li/ely
diagnosis?
A. Achalasia
". Diffuse esophageal spasm
. 7ncompetent lower esophageal sphincter
D. Eropharyngeal dysphagia
$. Scleroderma
$'planation(
)he correct answer is A. Achalasia is an ac.uired esophageal motility disorder that slowly develops. )he motility
is a-normal due to the loss of inhi-itory enteric neurons of the esophageal -ody and lower esophageal
sphincter. "oth vasoactive intestinal peptide and nitric o'ide function as inhi-itory neurotransmitters here* and
the presence of -oth is decreased in achalasia. Radiographs typically show a dilated esophagus that tapers at
the lower esophageal sphincter* producing a so-called 8-ird@s -ea/.8 "ecause of the poor motility* ingested food
is regurgitated and can lead to aspiration symptoms. <eart-urn can occur due to production of lactic acid in the
esophagus as the retained ingestate is fermented. %anometric demonstration of a-sent peristalsis in the
esophageal -ody and poor rela'ation of the lower esophageal sphincter with a swallow confirm the diagnosis.
)he primary complaint with diffuse esophageal spasm (choice ") is mid-sternal pain that can -e misdiagnosed
as cardiac pain. )he pain is caused -y prolonged contraction of the entire esophageal -ody. Symptoms can -e
-rought on -y eating certain hot or cold meals. A manometric study may show poor peristalsis in the smooth
muscle portion of the esophageal -ody* -ut lower esophageal sphincter function is unaffected.
)he primary complaint with incompetent lower esophageal sphincter (choice ) is heart -urn and regurgitation
due to gastroesophageal reflu'. $ndoscopic e'amination of the esophagus may reveal inflammation* erosions*
and even ulcers. A manometric study would show lower-than-normal resting tone in the lower esophageal
sphincter* or a sphincter that rela'es inappropriately.
)he fact that the patient@s symptoms do not occur until 9-> seconds after a swallow suggests that
oropharyngeal dysphagia (choice D) is not the diagnosis. )he presence of cough* hoarseness* or nasal
regurgitation commonly occurs with this disorder. Eropharyngeal dysphagia is often due to neurological or
muscle disorders li/e stro/e* amyotrophic lateral sclerosis* muscular dystrophy* or myasthenia gravis.
Scleroderma (choice $) is a connective tissue disease in which esophageal smooth muscle is gradually
replaced -y dense collagenous material. %anometry would show poor esophageal peristalsis and decreased
lower esophageal sphincter tone. Significant acid reflu' with resultant esophagitis is almost universal.
F9 D7A4AR%
A !9-year-old-woman is admitted to the hospital -ecause of syncopal attac/s and difficulty -reathing. She had
undergone mitral valve replacement # years ago. G-ray shows pulmonary congestion and an enlarged heart. A
phonocardiogram did not show a systolic murmur. 6ressure tracings from the aorta* left ventricle* and left atrium
are shown in the diagram. Which of the following diagnoses -est accounts for these findings?
A. Aortic regurgitation
". Aortic stenosis
. %itral regurgitation
D. %itral stenosis
$'planation(
)he correct answer is D. 7t is evident that the patient has mitral o-struction -ecause the left atrial pressure is
greater than the left ventricular pressure toward the end of diastole* when -lood is flowing from the left atrium into
the left ventricle. )he mitral valve replaced # years ago had undergone throm-osis resulting in o-struction of the
mitral orifice. )he very high left atrial pressure resulting from the throm-osed valve has caused pulmonary
edema* which accounts for the dyspnea.
)he aortic pressure and left ventricular pressure tracings are nearly superimposed during systole in the diagram*
which eliminates the possi-ility of aortic regurgitation (choice A) or aortic o-struction (choice ").
%itral regurgitation (choice D) is characteriAed -y a greatly elevated left atrial pressure toward the end of systole.
)he increase in pressure is caused -y -ac/ward flow of -lood from the left ventricle into the left atrium through
the lea/y mitral valve. )he lea/ occurs during systole* and is characteriAed -y a systolic murmur* which was not
noted in the patient. )he left atrial pressure is normal at the end of diastole with mitral regurgitation -ecause
-lood flows unimpeded from the atrium into the ventricle when the mitral valve is open.
A patient with complaints of somnam-ulism has fallen asleep. She passes from light sleep into a deeper sleep.
Bust -efore she e'periences an episode of somnam-ulism* her electroencephalogram is li/ely to show
A. alpha waves
". -eta waves
. delta waves
D. sleep spindles and 5-comple'es
$. theta waves
$'planation(
)he correct answer is . Delta waves are low-fre.uency* high-amplitude waveforms that herald the arrival of the
deepest type of non-R$% sleep* stage ! sleep. 7t is during this stage of sleep that somnam-ulism (sleepwal/ing)
occurs.
Alpha waves (choice A) are characteristic of rela'ed wa/efulness.
"eta-li/e activity (choice ") is characteristic of either alert wa/efulness or R$% sleep.
Sleep spindles and 5-comple'es (choice D) are characteristic of stage 9 sleep* which is a deeper sleep that
occupies roughly !:; of the sleep cycle.
)heta waves (choice $ ) are characteristic of light sleep (stage H).
A :H-year-old male smo/er presents with fever and a cough productive of greenish-yellow sputum. )he patient
states that he has had a morning cough with e'cessive mucus production for the past : years. Which of the
following a-normalities would most li/ely -e found in this patient?
A. Apical cavitary lesions on '-ray
". urschmann spirals in his sputum
. $levated salt levels in his sweat
D. $nlarged hilar lymph nodes on '-ray
$. 7ncreased Reid inde'
$'planation(
)he correct answer is $. )his patient presents with symptoms suggestive of acute infection (elevated
temperature* greenish-yellow sputum) on a -ac/ground of chronic -ronchitis* which is common in smo/ers.
<yperplasia and hypertrophy of mucous glands in chronic -ronchitis causes them to -e present at deeper
levels in the -ronchial wall than usual. )he ratio of the gland depth to the total thic/ness of the -ronchial wall is
termed the Reid inde'* which would -e increased in this patient.
Apical cavitary lesions (choice A) might -e indicative of cavitary tu-erculosis. )his condition is not associated
with e'cessive mucus production. <emoptysis and weight loss might also -e e'pected as clinical findings.
urschmann spirals (choice ") are found in asthmatic patients and represent mucus casts of small airways.
)his patient does not have the typical episodic history of acute asthmatic attac/s with acute dyspnea as the
ma+or clinical pro-lem.
$levated sodium chloride levels in sweat (choice ) are present in cystic fi-rosis. )his condition has an onset in
early life and is associated with e'cessive production of thic/ mucus* which predisposes to infection of the
airways and permanent damage.
$nlarged hilar lymph nodes (choice D) might suggest -ronchogenic carcinoma or a granulomatous process*
which would -e less li/ely than chronic -ronchitis. 7n addition* patients with carcinoma often present with
hemoptysis and weight loss* rather than e'cessive mucus production.
A neurological e'amination of a !I-year-old woman reveals a normal corneal refle' in her right eye* -ut no
consensual corneal refle' in her left eye. Which of the following additional findings might -e e'pected?
A. A-sence of pupillary light refle' of the left eye
". <yperacusis of the left ear
. 7na-ility to a-duct the right eye
D. =oss of pain and temperature of the left face
$. =oss of taste from the anterior two-thirds of the right tongue
0. 6tosis of the left eye
$'planation(
)he correct answer is ". )he first tric/ to this .uestion is to determine where the lesion is. )he corneal refle' is
tested -y touching a cotton wisp to the eye. A normal response would -e -lin/ing of the ipsilateral eye as well
as the contralateral eye (consensual refle'). )he afferent lim- of the corneal refle' is contained within the
ophthalmic division of the ipsilateral ophthalmic nerve (?H)* the efferent lim- is -y -oth (right and left) facial
nerves (?77). )his woman had a normal corneal refle' in her right eye* indicating a normal right ?H and right ?77.
<owever* she lac/ed a consensual refle'* indicating an a-normal left ?77th nerve. )he ne't tric/ to this .uestion
is to determine what other signs a lesion in the left ?77th nerve could produce. A lesion in the left ?77th would also
produce hyperacusis (increased sensitivity to sound) in the left ear -ecause of paralysis of the stapedius
muscle* which ordinarily dampens sound transmission through the middle ear.
)he a-sence of a pupillary light refle' of the left eye (choice A) could -e caused either -y a lesion of the left
optic nerve (& 773 afferent lim-) or -y a lesion of the left oculomotor nerve (& 7773 efferent lim-).
)he ina-ility to a-duct the right eye (choice ) could -e caused -y a lesion of the right a-ducens nerve (&
?7)* which innervates the lateral rectus muscle.
=oss of pain and temperature of the left face (choice D) could -e caused -y a lesion of the spinal nucleus of ?.
)his nucleus is located in the medulla* and receives pain and temperature information from the face via the
trigeminal nerve (& ?).
=oss of taste from the anterior two-thirds of the right tongue (choice $) could result from a lesion of the right &
?77.
6tosis of the left eye (choice 0) could result from a lesion of the left oculomotor nerve (& 777) -ecause of
denervation of the levator palpe-rae muscle. A lesion of the left ?77 would result in the ina-ility to close the left
eye.
A 9J-year-old woman with a history of irregular menses -ecomes amenorrheic. She had no pro-lems conceiving
her first child at the age of 9>* -ut she has -een trying unsuccessfully to -ecome pregnant for the past two
years. She also notes a weight gain of a-out > /g* increasing fatigue* puffy face and mar/ed cold intolerance. A
complete -lood count (") reveals a <- of HH.H and an %? of JC. 6hysical e'am reveals a moderate-siAed
diffuse enlargement of the thyroid gland. Which of the following thyroid profiles would most li/ely -e seen in this
woman?
A. =ow )>* low )!* high )S<
". =ow )>* low )!* low )S<
. =ow )>* high )!* low )S<
D. <igh )>* low )!* low )S<
$. <igh )>* high )!* low )S<
$'planation(
)he correct answer is A. )his woman is e'periencing signs and symptoms of hypothyroidism* the most common
cause of which is chronic thyroiditis* or <ashimoto@s thyroiditis. 7t is an autoimmune disorder* mostly affecting
women* in which antithyroid anti-odies are produced. )he immune response results in autoimmune destruction
of the thyroid gland* rendering it less a-le to produce thyroid hormone* causing hypothyroidism. Some other
symptoms of hypothyroidism are wea/ness* fatigue* coarse hair* constipation* hoarseness* and hearing loss.
Since the thyroid cannot produce thyroid hormone* -oth )> and )! would -e low. 7n trying to compensate for
low thyroid hormone levels* the pituitary gland releases e'cess )S< to stimulate the thyroid gland to ma/e more
thyroid hormone.
FI
)he wor/ diagrams in the figure a-ove show changes in left ventricular volume and pressure during one cardiac
cycle from a normal heart (diagram A) and following aortic valvular disease (diagram "). Which of the following is
e'pected to -e increased in the heart depicted in diagram " as compared to the normal heart depicted in diagram
A?
A. oronary artery o'ygen content
". oronary -lood flow during diastole
. oronary -lood flow during systole
D. oronary vein o'ygen content
$. %yocardial o'ygen tension (pE9)
$'planation(
)he correct answer is ". oronary -lood flow is regulated almost entirely -y the meta-olic re.uirements of the
cardiac muscle. )he heart depicted in diagram " has aortic stenosis. )he pea/ systolic pressure of the left
ventricle has increased from a normal value of a-out H9: mm <g to a-out HJC mm <g. )his increase in systolic
pressure has increased the stro/e wor/ output of the heart depicted -y diagram ". )he stro/e wor/ output is
e.ual to the area enclosed -y the volume-pressure diagram. )his increase in stro/e wor/ output increases the
o'ygen consumption of the heart* there-y decreasing the content of o'ygen in the venous effluent flowing from
the heart (choice D) as well as the o'ygen tension (pE9) in the myocardium (choice $). 7t should -e clear that
increasing myocardial o'ygen consumption will not affect the amount of o'ygen in the -lood (arterial o'ygen
content* choice A) entering the heart through the coronary arteries.
)he heart normally uses a-out IC; of the o'ygen in the arterial -lood flowing through the coronary circulation.
"ecause there is not much o'ygen left in the -lood* increases in -lood flow are re.uired to supply the heart with
additional amounts of o'ygen* i.e.* o'ygen e'traction cannot -e increased to a large e'tent. Ene other pro-lem is
that -lood flow falls to low levels during systole -ecause the coronary -lood vessels are compressed -y the
contracting muscle. )he increase in pea/ systole pressure caused -y aortic stenosis compresses the coronary
vessels even more than normal* causing systolic -lood flow to decrease greatly (choice ). )his decrease in
systolic flow coupled with the increase in myocardial o'ygen consumption that occurs with aortic stenosis causes
-lood flow to increase greatly during diastole.
A child is 9 standard deviations -elow the e'pected mean height for his age. <e also has delayed -one
maturation and a goiter. Analysis of genetic material reveals a point mutation in the thyroid hormone receptor.
Which of the following la-oratory results would -e e'pected in this patient?
A. Decreased radioactive iodine upta/e test (RA7D)
". Decreased resin )> upta/e test
. Decreased plasma )S< concentration
D. 7ncreased -asal meta-olic rate ("%R)
$. 7ncreased plasma )! concentration
$'planation(
)he correct answer is $. 4eneraliAed resistance to thyroid hormone is a rare genetic a-normality (Refetoff@s
syndrome). 7t results from mutations of the thyroid hormone receptor gene. Depending on the severity of the
disorder* patients may only -e mildly affected or may e'hi-it stri/ing hypothyroid-li/e symptoms including
decreased "%R (not increased* choice D). 4rowth can -e stunted* there may -e deaf mutism* and attention
span may -e short. "ecause the thyroid hormone resistance is generaliAed* the normal negative feed-ac/
effects of )! and )> at the hypothalamus and pituitary are also deficient. )his would lead to an increased
plasma )S< concentration (not decreased* choice ). "ecause of the increase in plasma )S<* iodine trapping
-y the thyroid follicular cells will -e increased* leading to an increase in RA7D (not decreased* choice A) and an
increase in serum )!. "ecause of the increase in serum )!* the e.uili-rium -etween )! and thyro'ine -inding
glo-ulin ()"4) will -e shifted toward increased -ound )! with a concomitant decrease in free )"4 -inding
sites. 7n the resin )> upta/e test* the added radioactive )> would thus preferentially -ind to the resin and not
the )"4* producing an increase in resin )> upta/e (not decrease* choice ").
A mor-idly o-ese (!:C l-) individual presents to the emergency department in respiratory distress. Arterial -lood
gas studies show a 6E9 of :: mm <g* a 6E9 of KC mm <g* and a p< of I.9#. hest G-ray films are
unremar/a-le* with no evidence of emphysema* tumor* fi-rosis* pulmonary infarction* or other disease.
Auscultation reveals a rapid -ut regular heart -eat* and the pulse is strong. Which of the following is the most
li/ely e'planation of the patient@s arterial hypo'emia?
A. Decreased capacity for pulmonary diffusion
". Decreased surface area of alveolar capillary mem-ranes
. <ypoventilation of central origin
D. <ypoventilation of peripheral origin
$. 7ne.ualities of ventilation and perfusion
$'planation(
)he correct answer is D. 7t is conceptually worth su-classifying hypo'emia in terms of the groups of the causes
listed in the answers. 7n this case* the patient has no evidence of primary pulmonary or cardiovascular disease*
and is /nown to -e mor-idly o-ese. %or-idly o-ese individuals are vulnera-le to the 6ic/wic/ian syndrome
(after a character in a Dic/ens novel)* in which pressure from a fatty nec/ causes intermittent airway
o-struction. )he many other causes of hypoventilation of peripheral origin include suffocation* su-mersion*
s/eletal a-normalities* trauma* phrenic nerve paralysis* polio* and tetanus.
auses for decreased capacity for pulmonary diffusion (choice A) of C9 include processes such as respiratory
distress syndrome* emphysema* pulmonary fi-rosis* and some granulomatous processes such as sarcoidosis.
auses for decreased surface area of alveolar capillary mem-ranes (choice ") include resection or
compression of the lung and emphysema.
)he usual cause of hypoventilation of central origin (choice ) is respiratory center depression -y morphine or
-ar-iturates.
auses for ine.ualities of ventilation and perfusion (choice $) include chronic -ronchitis* asthma* emphysema*
-ronchiectasis* some granulomatous processes* and tumors.
6ulmonary artery pressure
!:,9: mm <g
6ulmonary wedge pressure
>C mm <g
=eft ventricular pressure
H9C,: mm <g
Aortic pressure
H9C,#C mm <g
ardiac catheteriAation was performed on a :H-year-old woman -ecause of a J-month history of worsening
fatigue and shortness of -reath. What is the most li/ely diagnosis -ased on the pressures shown a-ove?
A. Aortic regurgitation
". Aortic stenosis
. %itral regurgitation
D. %itral stenosis
$'planation(
)he correct answer is D. )he pulmonary wedge pressure (which is used as an estimate of left atrial pressure) is
elevated to >C mm <g and the pulmonary artery pressure is elevated to !:,9: mm <g. )he left ventricular
end-diastolic pressure is normal -ut is not e.ual to the pulmonary wedge pressure. A pressure gradient of 9:
mm <g (>C - :) across the mitral valve is a clear indication of stenosis. )he fatigue and shortness of -reath
result from mild pulmonary edema caused -y the increase in pulmonary capillary pressure. Ene can surmise
that the pulmonary capillary pressure is elevated -ecause pressures are elevated at the arterial and venous
ends of the pulmonary circulation.
7n aortic regurgitation (choice A)* -lood flows -ac/ward through the aortic valve during diastole when the valve
is closed. =eft ventricular end-diastolic pressure (and pulmonary wedge pressure) may -e elevated with chronic
aortic regurgitation once the myocardium has failed* -ut aortic regurgitation itself will not result in a pressure
gradient across the mitral valve.
7n aortic stenosis (choice ")* the -lood is e+ected from the left ventricle into the aorta through a
smaller-than-normal opening. "ecause the resistance to e+ection of -lood is high* the left ventricular pressure
increases greatly with normal systolic pressure in the aorta.
%itral regurgitation (choice ) means -ac/ward flow of -lood through the mitral valve during systole. )his
accumulation of e'tra amounts of -lood in the left atrium during ventricular systole leads to an elevation in the
pulmonary wedge pressure (which is used as an estimate of left atrial pressure).
A patient comes in to the doctor -ecause of a chronic cough. <e notes occasional strea/s of -lood in his sputum.
hest '-ray reveals multinodular* cavitating lesions in the apical posterior segments of -oth lungs with evident
satellite lesions. )he condition descri-ed is li/ely to occur in the apices of the lungs -ecause they
A. are -etter perfused than the -ase
". are more acidic than the -ase
. contain more alveolar macrophages than the -ase
D. have a higher 6E9 than the -ase
$. ventilate -etter than the -ase
$'planation(
)he correct answer is D. )he presentation is typical for reactivation pulmonary tu-erculosis. )he patient may
also note fever* malaise* and weight loss. )he high 6E9 found in the upper portion of the lungs provides a
favora-le environment for growth of %yco-acterium tu-erculosis* leading to reactivation tu-erculosis. (7n
contrast* primary tu-erculosis tends to occur in the lower and middle lo-es* where small infectious particles are
most li/ely to lodge after -eing inhaled.)
?entilation increases from the top to the -ottom of the lung* so choice $ is wrong. 6erfusion increases even
more rapidly than ventilation* so choice A is also wrong. As a result* the ventilation-perfusion ratio decreases
from the top to the -ottom of the lung. )he higher ratio at the ape' of the lung results in a relatively elevated
6E9 at that location.
)he ape' of the lung has a higher p< than the -ase* so choice " is wrong. "ecause the ventilation-perfusion
ratio is higher at the ape'* 6E9 would -e lower* thus increasing the p<.
Regional differences in the density of alveolar macrophages (choice ) are not /nown to cause the descri-ed
predisposition.
A >:-year-old female* hospitaliAed after a motor vehicle accident* develops acute gastric stress ulcers. 7ncreases
in which of the following normal physiological parameters may have contri-uted to this condition?
A. "icar-onate transport
". $pithelial regenerative capacity
. %ucosal -lood flow
D. %ucus secretion
$. 6epsin production
$'planation(
)he correct answer is $. 6epsin production is a normal physiologic activity of the stomach that* in conditions of
stress* may overwhelm the stomach@s wea/ened defenses and result in gastric ulceration. 4astric acid
production is another condition that may increase and cause acute ulceration. 0urthermore* these two factors
may remain unchanged and still result in gastric ulcers if the gastric defenses are wea/ened -y stress. All of the
other choices represent normal defensive forces in the stomach.
7ncreased -icar-onate transport (choice A) would protect the gastric epithelium from the potentially harmful
acidity of the gastric contents. )he adherent mucus is relatively al/aline* providing local protection to the
superficial mucosa.
4astric epithelial cells can normally replicate rapidly* allowing mucosal defects to -e rapidly repaired. 7ncreasing
the regenerative capacity of the epithelium (choice ") would have a protective effect against ulceration.
)he gastric mucosa is richly supplied with -lood* providing the epithelial cells with an ample supply of nutrients*
o'ygen* and -icar-onate to contend with the harsh gastric microenvironment. Stress ulcers are associated with
compromised gastric -lood flow* not increased flow (choice ).
%ucus protects the gastric epithelium -y virtue of -eing water insolu-le* impermea-le to pepsin* and slowly
permeated -y acid (<1). 7ncreasing mucus production (choice D) has a protective effect for the gastric mucosa.
7n emphysema* the destruction of many alveolar walls changes the compliance of the respiratory system. Which
of the following clinical o-servations is directly related to this change in compliance?
A. "arrel chest
". hronic cough
. $'cessive mucus production
D. =ong* slow* deep -reathing pattern
$. 6in/ face
$'planation(
)he correct answer is A. A -arrel chest with increased anterior,posterior diameter is commonly o-served in
patients with long-standing* severe emphysema. )his change in chest shape occurs -ecause these patients*
who have high compliance of the lung proper* tend to function with their lungs to some degree 8over-inflated8
compared to people with normal lung compliance. )his over-inflation limits their a-ility to ta/e further deep
-reaths. ()he 8-alloon8 of emphysematous lung remains compliant* -ut the 8-o'8 of the chest wall is not very
compliant and limits the volume of air that can -e inhaled). 6atients with moderately severe emphysema are
a-le to maintain an ade.uate lung ventilation -y ta/ing many short -reaths (compare with choice D)3 this
physiology is sometimes e'pressed -y descri-ing these patients as 8pin/ puffers8 (choice $).
hronic cough (choice ") in emphysema patients is not directly related to the change in compliance.
$'cessive mucus production (choice ) is more characteristic of chronic -ronchitis than of emphysema.
A :!-year-old alcoholic presents with complaints of tremors and muscle twitching. 6hysical e'amination reveals
the presence of )rousseau@s sign. =a-oratory data show that serum magnesium is L H m$.,= (normal* H.! - 9.9
m$.,=). Which of the following findings would -e most consistent with this information?
A. Decreased serum calcium
". Decreased serum phosphate
. 7ncreased -one density
D. 7ncreased plasma parathyroid hormone concentration
$. 7ncreased urinary cA%6 concentration
$'planation(
)he correct answer is A. %alnutrition associated with chronic alcoholism can lead to a severe magnesium
deficiency. )he effect of low serum magnesium on parathyroid hormone secretion (6)<) depends on severity
and duration. An acute decrease in serum magnesium will increase 6)< secretion* while a prolonged severe
deficiency results in decreased 6)< secretion. )here is also evidence that the action of 6)< is decreased with
chronic magnesium deficiency. <ence* this patient is suffering from 8functional8 hypoparathyroidism. )he low
serum calcium can produce wea/ness* tremors* muscle fasciculations* and seiAures. A positive )rousseau@s
sign indicates the presence of latent tetany. 7t is o-served -y inflating a -lood pressure cuff a-ove systolic
-lood pressure for at least 9 minutes. A positive reaction consists of the development of carpal spasm* with
rela'ation occurring within seconds after deflating the cuff. 7n patients with magnesium deficiency* magnesium
administration will produce a prompt rise in plasma 6)< with su-se.uent restoration of serum calcium
concentration to normal.
With functional hypoparathyroidism -one density would -e decreased (not increased* choice ).
)he com-ination of decreased 6)< secretion (not increased* choice D) and decreased effectiveness of 6)<
produce hypocalcemia and hyperphosphatemia (not hypophosphatemia* choice ").
Drinary cA%6 would pro-a-ly -e decreased (not increased* choice $)* given the low 6)<.
A child who has had a-normal development of the mem-ranous -ones has a -road s/ull with associated facial
and dental anomalies. Which other -ones are most li/ely to also -e affected?
A. lavicles
". 0emurs
. %etatarsals
D. 6halanges
$. )i-ias
$'planation(
)he correct answer is A. 7n a syndrome called cleidocranial dysostosis* a-sence of part of the clavicles
accompanies a -road s/ull* and facial and dental anomalies. &ote that you could also have answered this
.uestion -y noting that of the -ones listed* only the clavicles form -y intramem-ranous ossification.
)he femurs (choice ")* metatarsals (choice )* phalanges (choice D)* and ti-ias (choice $) are cartilaginous
(formed -y endochondral ossification) rather than mem-ranous -ones.
A :C-year-old man presents to his doctor with diarrhea* flushing* and wheeAing. 6hysical e'amination is
significant for a grade 77,?7 diastolic murmur located at the right sternal -order at the !th intercostal space. Which
of the following su-stances is most li/ely to -e elevated in this patient@s urine?
A. :-<7AA
". <?A
. 6henylalanine
D. Selegiline
$. ?anillylmandelic acid (?%A)
$'planation(
)he correct answer is A. :-<7AA is a meta-olite of serotonin* a ma+or secretory product of carcinoid tumors. )he
signs and symptoms of carcinoid syndrome include diarrhea* flushing* and wheeAing. )he cardiac a-normalities
are commonly concentrated in the right heart -ecause carcinoid secretory products are degraded or deto'ified
in the lung.
<?A (choice ") is a -rea/down product of dopamine through the %AE or E%) meta-olism pathways.
6henylalanine (choice ) is an essential amino acid that is used to synthesiAe tyrosine* the precursor of the
catecholamines (dopamine* norepinephrine* and epinephrine).
Selegiline (choice D) is a %AE-" inhi-itor that inhi-its the degradation of dopamine. 7t is used in the treatment
of 6ar/inson@s disease.
?%A (choice $) is a meta-olite of epinephrine that is elevated in the urine of individuals with
pheochromocytoma.
A 9K-year-old male is -rought to a physician -ecause of a head in+ury. <is wife states that she and her hus-and
were wal/ing on the sidewal/ when he suddenly fell to the ground and hit his head. She said similar episodes had
occurred -efore* -ut this was the first time that he had -een in+ured. ardiovascular evaluation is unrevealing. An
electroencephalogram administered at the appropriate time would pro-a-ly reveal which of the following types of
seiAures?
A. A-sence
". Atonic
. %yoclonic
D. )onic
$. )onic-clonic
$'planation(
)he correct answer is ". Atonic or 8drop8 seiAures are characteriAed -y a sudden loss of postural muscle tone
that lasts only a few seconds. Although consciousness may -e impaired -riefly* there is rarely postictal
confusion. A very -rief seiAure may cause only a drop of the head* -ut a longer seiAure may cause the patient
to slump to the ground. )his type of seiAure may -e .uite dangerous -ecause of the ris/ of head in+ury with a
sudden fall. Drugs prescri-ed for this condition include valproic acid* clonaAepam* fel-amate* viga-atrin* and
lamotrigine.
A-sence seiAures (choice A)* also /nown as petit mal seiAures* are characteriAed -y -lan/ stares and an
a-sence of any change in position. )hey typically occur in children. Drugs used in this disorder include
ethosu'imide* valproic acid* and clonaAepam.
%yoclonic seiAures (choice ) are characteriAed -y sudden* -rief muscle +er/s that may involve part of the -ody
or the whole -ody. A physiologic form of myoclonus occurring in healthy individuals is the sudden +er/ing
movement that sometimes occurs while falling asleep. Drugs used for this condition include valproic acid and
clonaAepam.
)onic seiAures (choice D) are characteriAed -y the sudden onset of sustained a'ial and lim- muscle contraction.
6ure tonic seiAures tend to occur in children.
)onic-clonic seiAures (choice $)* also /nown as grand mal seiAures* are characteriAed -y an initial phase with
tonic contraction of muscles throughout the -ody. After a short time* the seiAure evolves into the clonic phase*
in which periods of muscle rela'ation are superimposed on muscle contraction. )here is a significant postictal
phase. Drugs used for this disorder include car-amaAepine* phenytoin* and valproic acid.
A cyanotic infant is discovered to have a ventricular septal defect* an overriding aorta* right ventricular
hypertrophy* and complete pulmonic stenosis. Which of the following accompanying congenital anomalies
permits survival in this patient?
A. "icuspid aortic valve
". Estium secundum defect
. 6atent ductus arteriosus
D. 6atent foramen ovale
$. 6reductal coarctation of aorta
$'planation(
)he correct answer is . )he ductus arteriosus connects the aorta with the pulmonary artery. 7f it remains
patent after -irth* it allows o'ygenated -lood to flow from the aorta to the pulmonary artery. 7n this patient with
tetralogy of 0allot with complete right ventricular outflow o-struction* this anastomosis is a crucial source of
-lood to the pulmonary vasculature.
A -icuspid aortic valve (choice A) may -e asymptomatic -ut can lead to infective endocarditis* left ventricular
overload* and sudden death. 7t is a common cause of aortic stenosis. 7t would not -enefit a patient with
tetralogy of 0allot in any way.
Estium secundum defect (choice ") is the most common form of atrial septal defect (ASD). ASD is an acyanotic
congenital heart disease that would not improve cardiovascular function in a patient with tetralogy of 0allot.
A patent foramen ovale (choice D) is a slit-li/e remnant of communication -etween the left and right atria in the
fetus. 7t is usually not of clinical significance.
A preductal coarctation of the aorta (choice $) involves narrowing of the aorta pro'imal to the opening of the
ductus arteriosus. )his would prevent ade.uate -lood flow through a possi-le life-preserving 6DA and would
result in the patient@s demise.
A H!-year-old male presents with type 7 dia-etes mellitus. <is mother wants to /now if the -oy@s -rother might also
have an increased ris/ of getting the disease. Which of the following genotypes* if present in the -rother* would
-e associated with the greatest ris/ of developing dia-etes?
A. "9I,"9I
". DR9,DR9
. DR9,DR!
D. DR>,DR>
$. DR>,DR!
$'planation(
)he correct answer is $. A heteroAygous individual with <=A-DR> and <=A-DR! has a >>-fold greater relative
ris/ for developing dia-etes than individuals without these two <=A antigens.
<omoAygous individuals with <=A-"9I (choice A) are more li/ely to develop an/ylosing spondylitis and Reiter@s
disease.
6atients who are homoAygous for <=A-DR9 (choice ") have a reduced ris/ of dia-etes mellitus. 7ndividuals with
the <=A-DR! allele have an increased ris/ of dia-etes. <eteroAygotes for <=A-DR9 and <=A-DR!(choice )
li/ely have an intermediate ris/.
7ndividuals who are positive for the <=A-DR> antigen (choice D) have three times the ris/ of developing
dia-etes mellitus.
A patient hospitaliAed with pneumonia has a thyroid hormone panel ordered along with other routine -lood wor/.
Serum )> is decreased* -ut serum )! and )S< are within the normal range. 0rom this information* the physician
concludes that the patient
A. has low )> syndrome (euthyroid sic/ syndrome)
". has primary hypothyroidism
. should -e treated with thyro'ine replacement
D. will also e'hi-it decreased serum reverse )> concentration
$. will also e'hi-it increased activity of :@-monodeiodinase in peripheral tissues
$'planation(
)he correct answer is A. )he low )> syndrome or 8euthyroid sic/ syndrome8 occurs with certain systemic
illnesses li/e pneumonia or septicemia* after ma+or surgery* and with malnutrition or starvation. Whatever the
cause* the syndrome is characteriAed -y a decrease* not increase (choice $)* in :@-monodeiodinase activity in
peripheral tissues li/e liver and /idney. As a conse.uence the conversion of circulating )! to )> is impaired and
-lood levels of )> decrease. )he decrease in serum )> is thought to -e a protective adaptation to decrease
cata-olic processes during the illness or shortage of energy su-strates.
6rimary hypothyroidism (choice ") is characteriAed -y decreased serum )! and increased serum )S<. 7n the
euthyroid sic/ syndrome* serum )! and )S< are usually within the normal range.
"ecause the low )> is pro-a-ly a protective adaptation* treatment with thyro'ine replacement (choice ) is of
little -enefit and may actually -e harmful. Ence the patient recovers from the illness or malnutrition* serum
levels of )> gradually return to normal on their own.
A normal step in the -rea/down of reverse )> also involves the action of :@-monodeiodinase* which converts
reverse )> to >*>@-diiodothyronine. <ence* with the low )> syndrome* serum concentration of reverse )> is
increased* not decreased (choice D)* due to decreased -rea/down.
A !#-year-old male is seen for persistent edema. 7nitial la-oratory studies indicate that he has nephrotic
syndrome* and a renal -iopsy is diagnostic of mem-ranous glomerulonephritis. Which of the following su-stances
will -e elevated in the plasma in this patient?
A. Al-umin
". Ammonia
. holesterol
D. 4lucose
$. 6otassium
$'planation(
)he correct answer is . )he nephrotic syndrome descri-es a group of la-oratory findings associated with
glomerular diseases which share the common characteristic of 8lea/y glomeruli.8 =arge -iochemicals* normally
una-le to cross out of glomerular capillaries into "owman@s space* are lost into the urine. Serum proteins are
lost in large .uantities and can -e detected as -oth hypoproteinemia and massive proteinuria. Al-umin (choice
A)* a relatively small plasma protein (%W KK*CCC) is lost very readily* leading to hypoal-uminemia. Serum
concentrations of small compounds such as potassium (choice $) and glucose (choice D)* which are highly
permea-le in the normal glomerulus* are unaffected -y glomerulonephritides producing the nephrotic
syndrome. Similarly* -lood urea nitrogen is unaffected* and serum ammonia levels (choice ") are unchanged.
)he final component of the nephrotic syndrome (-esides hypoproteinemia* hypoal-uminemia and massive
proteinuria) is hyperlipidemia. 7t is apparently a function of -oth increased hepatic fat synthesis and decreased
fat cata-olism. 7ncreased cholesterol (choice )* triglycerides* and lipoproteins are found in serum in
mem-ranous glomerulonephritis* and these lipids lea/ into the urine* producing lipiduria.
A patient with mild congestive heart failure is treated with high-dose furosemide and diureses 9: pounds of fluid.
A complete -lood count (") ta/en -efore the diuresis shows an R" count of ! million,mm>3 a " ta/en
after diuresis shows a R" count of I million,mm>. Which of the following is the most li/ely e'planation?
A. yanotic heart disease
". 7ncreased erythropoietin
. 6olycythemia vera
D. Relative polycythemia
$. Renal cell carcinoma
$'planation(
)he correct answer is D. )his is an e'ample of relative polycythemia* in which there is an increased hematocrit
or R" count without a true increase in the total num-er of -ody R"s. What usually happens in these cases
is a significant reduction in plasma volume due to processes such as dehydration* vomiting* diarrhea* or
diuresis.
yanotic heart disease (choice A)* via appropriate erythropoietin secretion* can cause secondary a-solute
polycythemia.
7ncreased erythropoietin (choice ")* whether appropriately or inappropriately secreted* can cause secondary
a-solute polycythemia.
6olycythemia vera (choice ) causes primary a-solute polycythemia with usually low erythropoietin levels.
Renal cell carcinoma (choice $)* via inappropriate erythropoietin secretion* can cause secondary a-solute
polycythemia.
An elderly female with a history of alcoholic disease develops +aundice and mar/ed anasarca. Which of the
following is the most li/ely pathophysiology of her persistent edema?
A. =ymphatic o-struction
". Reduced central venous pressure
. Reduced plasma oncotic pressure
D. Sodium retention
$. ?enous throm-osis
$'planation(
)he correct answer is . <epatic failure occurring in cirrhosis reduces the capacity of the liver to synthesiAe
sufficient .uantities of plasma proteins (mostly al-umin) necessary to maintain plasma oncotic pressure. =ow
plasma oncotic pressure allows fluid from the intravascular fluid component to move into the interstitial space*
producing plasma volume contraction and edema.
=ymphatic o-struction (choice A) occurs as a result of mechanical -loc/age of lymphatics -y tumor*
inflammatory processes* or certain parasitic infections. irrhosis does not lead to lymphatic o-struction.
Reduced central venous pressure (choice ") does not cause edema. onversely* increased central venous
pressure* which may arise with congestive heart failure* throm-osis* or cirrhosis can lead to increased
hydrostatic pressure and edema.
Sodium retention (choice D) is an important cause of edema in patients with poor renal perfusion. )he /idneys
respond -y retaining sodium and increasing plasma volume in an effort to increase renal -lood flow. Sodium
retention in cirrhosis is secondary to the decrease in plasma oncotic pressure and conse.uent decrease in
plasma volume.
?enous throm-osis (choice $) can lead to edema3 however* the diminished synthesis of coagulation proteins in
cirrhosis predisposes to -leeding* not throm-osis.
6hysical e'amination of a neonate is remar/a-le for a holosystolic murmur. )here is no cyanosis.
$chocardiography demonstrates an ostium primum defect in the lower part of the interatrial septum that is
accompanied -y malformations of the ad+acent atrioventricular valves. )hese lesions are most li/ely associated
with which of the following disorders?
A. ystic fi-rosis
". Down syndrome
. 4aucher disease
D. %arfan syndrome
$. )urner syndrome
$'planation(
)he correct answer is ". )he most common type of atrial septal defect is the ostium secundum type. hildren
with Down syndrome* however* are fre.uently afflicted with the ostium primum type of atrial septal defects*
which may -e accompanied -y tricuspid and mitral valve malformations. %ore comple' atrioventricular septal
defects may also occur in this disorder. hildren e'hi-iting these lesions should -e specifically evaluated for
chromosomal a-normalities. linically* the lesions produce left-to-right shunts with late cyanosis (after the right
ventricle hypertrophies in response to developing lung disease from the increased -lood flow in the pulmonary
system).
&either cystic fi-rosis (choice A) nor 4aucher disease (choice ) is specifically associated with cardiovascular
defects.
Dissecting aortic aneurysm is associated with %arfan syndrome (choice D).
)urner syndrome (choice $) is associated with coarctation of the aorta.
An H#-year-old male comes to the university clinic supported -y his roommates -ecause he cannot wal/. <e
descri-es a rapidly evolving wea/ness affecting his legs and feet starting 9 days ago. En physical e'amination he
cannot move his feet or an/les and he can -arely raise his thighs off the -ed. <e has symmetrical hyporefle'ia of
the legs* -ut his sensorium is completely intact. Scanning his chart* the physician notes that he was treated HC
days previously for an upper respiratory tract infection. )he immunological response producing the patient@s
symptoms is most intense at which of the following locations?
A. =ateral corticospinal tracts
". &euromuscular +unction
. 6recentral gyrus
D. S/eletal muscles
$. Spinal motor nerves
$'planation(
)he correct answer is $. )he patient has developed 4uillain-"arrM syndrome* also /nown as inflammatory
polyneuropathy. )his presentation is classic-rapidly evolving lim- wea/ness with symmetrical hyporefle'ia -ut
normal sensation. )he syndrome fre.uently follows viral infections and may evolve into complete paralysis with
respiratory failure. 4uillain-"arrM syndrome is thought to -e an autoimmune disease. )he clinical course is
correlated with a chronic inflammatory infiltrate and demyelination of peripheral nerves* especially spinal and
cranial motor nerve roots.
7nflammation localiAed to a small portion of spinal cord (lateral corticospinal tracts3 choice A) or cere-ral corte'
(precentral gyrus3 choice ) may occur in progressive multifocal leu/oencephalopathy (6%=) or in multiple
sclerosis (%S). 6%= occurs in the immunosuppressed* and %S presents with hyperrefle'ia (upper motor neuron
signs).
)he classic autoimmune disease involving the neuromuscular +unction (choice ") is myasthenia gravis. Although
the wea/ness caused -y myasthenia gravis may affect the legs* e'traocular muscles are involved in the ma+ority
of cases* and isolated lim- wea/ness is rare.
Diseases primarily affecting the s/eletal muscle (choice D) include autoimmune inflammatory myopathies such
as dermatomyositis and polymyositis* which typically affect the pro'imal lim- muscles more than the distal
musculature. 7nclusion -ody myositis is a slowly developing disease that is asymmetrical and occurs in older
individuals.
A I!-year-old woman with type 77 dia-etes mellitus* hypertension* and end-stage renal failure has -een
dialysis-dependent for several years. She develops a fracture of the left femoral head. )issue ta/en from the
fracture at the time of internal fi'ation is noted to have increased osteoclastic activity* with nota-le tunnel-li/e
dissection -y osteoclasts into the -ony tra-eculae. 7ncreased levels of which of the following hormones is most
li/ely to -e responsi-le for this lesion?
A. alcitonin
". ortisol
. $rythropoietin
D. 4lucagon
$. 6arathyroid hormone
$'planation(
)he correct answer is $. )he patient has developed hyperparathyroidism* a well-recogniAed se.uela of chronic
renal failure. )his condition is caused -y high levels of serum phosphate and low serum calcium* which
stimulate the release of parathyroid hormone (6)<) in an effort to normaliAe the calcium,phosphate ratio. 6)<
stimulates osteo-lasts to -ecome osteoclasts* which dissolve the -one reservoir of calcium and release it into
the -lood. )his condition* /nown as renal osteodystrophy* may lead to osteomalacia and osteitis fi-rosa cystica*
which is classically associated with dissecting osteitis* as descri-ed in this .uestion.
alcitonin (choice A)* which serves to lower serum calcium levels* is produced in small .uantities in chronic
renal failure* as serum calcium is already pathologically low.
$'cess cortisol (choice "3 ushing@s syndrome) may produce osteoporosis and pathologic fractures* -ut there
is no relationship -etween chronic renal failure and cortisol e'cess. ushing@s syndrome is generally secondary
to adrenal or pituitary adenomas or primary adrenal hyperfunction.
$rythropoietin (choice ) production is deficient in chronic renal failure. $'cess erythropoietin activates
erythrocyte precursors* -ut does not activate osteoclasts.
4lucagon (choice D) e'cess is a very rare entity* occurring in a minority of islet cell tumors* and is not
associated with renal failure. <igh glucagon produces a transitory s/in rash* anemia* and a form of dia-etes
mellitus.
A 9:-year-old male reports episodic 8spells8 characteriAed -y palpitations* sweating* nervousness* and feelings of
an'iety. En e'amination* the man@s -lood pressure is HK:,J: mm <g. 6lasma norepinephrine is !:C pg,m=
(normal* H:C-!CC pg,m=)* plasma epinephrine is HH: pg,m= (normal* 9:-HCC pg,m=)* and 9!-hour urinary ?%A is
HH mg (normal* L # mg). Which of the following is the most li/ely cause of the patient@s hypertension?
A. HH--eta-hydro'ylase deficiency
". onn@s syndrome
. 6heochromocytoma
D. Renin-secreting tumor
$. Dnilateral renal artery stenosis
$'planation(
)he correct answer is . A pheochromocytoma is a tumor arising from chromaffin cells that secretes e'cess
catecholamines (norepinephrine* epinephrine* or -oth). 7t is one endocrine cause of hypertension* due to
peripheral vasoconstriction and,or increased cardiac output. While most patients have higher than normal
-aseline plasma levels of catecholamines* it is not uncommon for paro'ysmal symptomatic episodes to -e
superimposed upon the -asal pro-lem. )hese 8attac/s8 may occur several times a wee/ (or more often) and
last for up to H: minutes. During an attac/* respiration can increase* the patient may -ecome aware of a forceful
pounding of the heart that progresses to include a thro--ing headache* and peripheral vasoconstriction can
raise -ody temperature and lead to refle' sweating. %ar/ed an'iety may also accompany the episode.
Diagnosis can -e confirmed -y measuring increased plasma or urinary catecholamines or their meta-olites.
HH--eta-hydro'ylase deficiency (choice A) is a congenital disorder than can cause hypertension due to
e'cessive production of the wea/ mineralocorticoid* deo'ycorticosterone* -y the inner two Aones of the adrenal
corte'. )his results in e'cessive renal retention of sodium and water and su-se.uent hypertension. While the
hypertension is usually present from -irth* a late-onset variant of this disorder has -een descri-ed in which the
symptoms do not present until late childhood or adolescence. ?iriliAation is also present due to e'cessive
secretion of adrenal androgens.
onn@s syndrome (choice ") is another endocrine cause of hypertension. 7n this case* the increase in -lood
pressure is due to e'cessive renal retention of sodium and water resulting from the increased plasma
concentration of aldosterone. )he paro'ysmal symptoms and increased catecholamines present in this patient
are not present with primary hyperaldosteronism.
Renin-secreting tumors (choice D) are rare and can -e confused with primary hyperaldosteronism. )he
e'cessive secretion of renin -y the tumor can increase the formation of angiotensin 77 with su-se.uent
hyperaldosteronism. Sodium and water retention* together with hypo/alemia* are present. 7ncreased plasma
renin and plasma aldosterone are suggestive of a renin-secreting tumor* whereas primary hyperaldosteronism
would present as increased plasma aldosterone* -ut decreased plasma renin.
Dnilateral renal artery stenosis (choice $) produces an angiotensin 77-dependent form of hypertension.
Decreased renal perfusion* often due to atherosclerosis or fi-romuscular hyperplasia of the renal arteries*
results in increased renin secretion and su-se.uently increased plasma angiotensin 77. )he resultant increase in
aldosterone secretion and arteriolar vasoconstriction contri-utes to the hypertension.
A I!-year-old woman* in otherwise good health* tripped and in+ured her right leg 9 days previously and has -een
-edridden since the accident. )wo hours ago* she -ecame delirious. En physical e'amination* her temperature is
JJ 0* -lood pressure is H9C,IC mm <g* heart rate is HHC* and respiratory rate is >9. 6ulse o'imetry shows an
o'ygen saturation of #C;* and a chest '-ray film is normal. Which of the following is the most li/ely diagnosis?
A. Acute cere-ral hemorrhage
". Acute cere-ral infarction
. %yocardial infarction
D. 6ulmonary infarction
$. 6ulmonary throm-oem-olism
$'planation(
)he correct answer is $. <ip fracture and prolonged -ed rest are classic ris/ factors for the development of
pulmonary throm-oem-oli (6$). ommon clinical manifestations of 6$ are hypo'ia (due to ventilation,perfusion
mismatch) despite a normal chest '-ray* tachycardia* and delirium in older patients.
ere-ral hemorrhage (choice A) might cause delirium -ut would not directly cause hypo'ia unless the patient
was hypoventilating (e.g.* -ecause of -rainstem involvement).
ere-ral infarction (choice ") could produce delirium -ut would not directly cause hypo'ia unless the patient
was hypoventilating.
%yocardial infarction (choice ) could account for delirium and tachycardia* -ut not for hypo'ia with a normal
chest '-ray. Severe congestive heart failure after myocardial infarction could cause hypo'ia due to pulmonary
edema* -ut the chest '-ray would not -e normal.
6ulmonary infarction (choice D) may cause delirium* tachycardia* and hypo'ia* -ut the chest '-ray may -e
a-normal. A chest '-ray performed within H9->K hours after a pulmonary infarct may reveal a peripherally
located* wedge-shaped infiltrate.
A 9K-year-old woman with sic/le cell anemia develops pneumonia* and as a complication* develops very painful
focal ischemia in the muscles and +oints. Which part of the renal vasculature would -e most vulnera-le to similar
damage during this sic/le cell crisis?
A. Arcuate arteries
". 4lomerular capillaries
. 7nterlo-ular arteries
D. Renal artery
$. ?asa recta
$'planation(
)he correct answer is $. Sic/ling crises can -e triggered -y hypo'ia caused -y high altitude or pneumonia. )he
very high osmolarity of the renal medulla particularly favors sic/ling of erythrocytes in the vasa recta. )he
resultant ischemia can cause a patchy papillary necrosis* proteinuria* and sometimes* cortical scarring.
=arger -lood vessels such as the arcuate arteries (choice A)* interlo-ar arteries (choice )* and renal arteries
(choice D) are usually not occluded -y sic/led cells.
)he glomerular capillaries (choice ") are small enough to -e occluded -y sic/led cells* -ut they are well
o'ygenated and the -lood within them is not hypertonic* so the glomerular capillaries are significantly less li/ely
to -ecome occluded then the vasa recta.
7n retinitis pigmentosa* 8night--lindness8 is an early symptom of visual loss. Which of the following e'plains this
phenomenon?
A. ones are relatively preserved compared to ganglion cells
". ones are relatively preserved compared to rods
. 4anglion cells are relatively preserved compared to cones
D. 4anglion cells are relatively preserved compared to rods
$. Rods are relatively preserved compared to cones
$'planation(
)he correct answer is ". Retinitis pigmentosa is a familial degenerative disease of the retina that most often has
recessive genetics. )he initial pro-lem appears to -e alterations in the pigmented epithelium -elow the retina*
particularly in the most anterior portions of the retina. )he pigmented epithelial cells -ecome disrupted and lea/
pigment* which accumulates along the attenuated -lood vessels (and can -e seen with an ophthalmoscope). A
conse.uence of this damage is that the rod cells that are normally nutritionally supported and 8groomed8 -y the
pigment epithelium also undergo degenerative changes. Since the cones are relatively preserved* day vision is
preserved* -ut night vision* which is highly dependent on rods* -egins to decay. With disease progression*
vision is completely or nearly completely lost and the retina -ecomes so distorted that only a single row of cone
nuclei with scattered stumpy cone remnants is all that remains of the photoreceptor layer.
A >9-year old women complains of amenorrhea since delivery of a -a-y H: months previously* despite the fact
that she did not -reast feed her -a-y. )he delivery was complicated -y e'cessive hemorrhage that re.uired
transfusion of 9.: liters of -lood. She has also -een fatigued and has gained an additional HC pounds since the
-a-y was -orn. =a-oratory data show the following(
Serum =< L H 7D,= (normal* !-9! 7D,=)
Serum estradiol : pg,m= (normal* 9C - HCC pg,m=)
Serum )S< C.H mD,= (normal* C.: - : mD,=)
Serum 4< > ng,m= (normal* L : ng,m=)
Serum A)< 9# pg,m= (normal* HC - :C pg,m=)
Serum prolactin 9 ng,m= (normal* L9C ng,m=)
7n+ection of :CC Ng of )R< failed to produce the e'pected rise in -oth serum )S< and prolactin. Which of the
following diagnoses most li/ely e'plains the findings in this patient?
A. <ashimoto@s thyroiditis
". 7solated gonadotropin deficiency
. 6rimary amenorrhea
D. 6rolactinoma
$. Sheehan@s syndrome
$'planation(
)he correct answer is $. Sheehan@s syndrome is hypopituitarism due to ischemic damage to the pituitary
resulting from e'cessive hemorrhage during parturition. )he pituitary is enlarged during pregnancy3 it is more
meta-olically active* and more suscepti-le to hypo'emia. 0urthermore* the -lood vessels in the pituitary may -e
more suscepti-le to vasospasm -ecause of the high estrogen. 7n a-out >C; of women who hemorrhage
e'cessively during parturition* some degree of hypopituitarism eventually -ecomes manifest. )he symptoms
depend on how much of the pituitary is damaged and what cell types are destroyed. )he patient descri-ed
a-ove e'hi-ited persistent amenorrhea after delivery of her infant. )his is due to destruction of pituitary
gonadotrophs and diminished secretion of gonadotropins (=<). )here also appears to have -een significant
destruction of lactotrophs since )R< in+ection failed to induce an increase in prolactin. <ad the women
attempted to -reast-feed her infant* a failure to lactate mostly li/ely would have occurred. )his case is also
characteriAed -y secondary hypothyroidism. )he low )S< and failure to respond to )R< in+ection is
confirmatory. orticotrophs appear to have -een spared since plasma A)< is normal. 7t is not clear whether
somatotrophs were damaged. 0urther testing would -e needed to see if 4< reserve is diminished.
<ashimoto@s thyroiditis (choice A) is an autoimmune disorder that produces primary hypothyroidism. "ecause of
diminished negative feed-ac/ effects of )!* serum )S< is usually increased (not decreased).
7solated gonadotropin deficiency (choice ") produces amenorrhea and is associated with low serum =< and
estradiol. <ypogonadotropic hypogonadism can occur in female athletes that over-train* in anore'ia nervosa* in
o-esity* or with other emotional or physical stresses. <owever* other pituitary hormones are unaffected (-y
definition isolated gonadotropin deficiency only involves a decrease in gonadotropins).
6rimary amenorrhea (choice )* -y definition* means failure of menstrual cycles to ever -egin. Since this woman
has delivered a -a-y* primary amenorrhea is highly unli/ely.
A prolactinoma (choice D) is a functional pituitary tumor that secretes e'cessive prolactin. )his can cause
amenorrhea -y suppressing the 4nR<-pituitary-gonad a'is. )he patient descri-ed a-ove has decreased
prolactin secretion.
A !C-year-old man presents to his physician with complaints of su-sternal pain radiating to his left shoulder*
occurring when he is at rest. )he pain improves when he gets up and moves around* and responds to su-lingual
nitroglycerin. )his patient@s symptoms are thought to -e caused -y which of the following processes?
A. oronary artery atherosclerosis
". oronary artery em-olism
. oronary artery spasm
D. oronary artery throm-osis
$. oronary artery vasculitis
$'planation(
)he correct answer is . )he pattern of angina descri-ed is called 6rinAmetal@s variant angina* and is -elieved
to -e due to coronary artery spasm occurring during rest. Dp to I:; of patients with this disorder can -e shown
to have a fi'ed o-struction within H cm of the site of arterial spasm. 6atients with 6rinAmetal@s angina are often
younger than patients with unsta-le angina secondary to coronary atherosclerosis. )he diagnosis may -e
confirmed -y o-serving a spontaneous coronary artery spasm (or provo/ing one -y administering ergonovine
or acetylcholine* or instructing the patient to hyperventilate) during angiography.
Atherosclerosis (choice A) is the cause of typical angina that occurs with e'ertion.
$m-olism (choice ") in the coronary arteries is somewhat unusual* -ut can occur if a pla.ue from the aorta
-rea/s off and lodges in a coronary vessel.
)hrom-osis (choice D) characteristically produces unsta-le or crescendo angina with worsening chest pain.
?asculitis (choice $) can uncommonly cause angina* nota-ly as part of 5awasa/i@s disease.
A 9H-year-old woman attempted suicide -y ta/ing an overdose of -ar-iturates. En arrival in the emergency
department* her -lood pressure is J:,K: and her pulse is HC: per minute. )he physician in the intensive care
unit orders arterial -lood gases. Which of the following values would you e'pect in this patient?
A. 6E9 O !:* 6E9 O !:* p< O I.!:
". 6E9 O ::* 6E9 O IC* p< O I.:C
. 6E9 O K:* 6E9 O >:* p< O I.!:
D. 6E9 O I:* 6E9 O KC* p< O I.>C
$. 6E9 O J#* 6E9 O KC* p< O I.9C
$'planation(
)he correct answer is D. "ar-iturate overdose causes respiratory depression* resulting in car-on dio'ide
retention (producing increased 6E9 and decreased p<) and hypo'emia (decreased 6E9). 7n other words* the
patient has respiratory acidosis. Pou should loo/ for a low 6E9* high 6E9* and acidotic p<. hoices A* "* *
and $ do not fulfill these re.uirements. &ote that choice might -e e'pected in a patient who is
hyperventilating to the point of respiratory al/alosis( diminished E9 (the usual drive for hyperventilation in
nonpsychiatric hyperventilation)* diminished E9* and mildly al/alotic p<.
A -a-y is -orn prematurely at 9# wee/s gestational age with a -irthweight of H9CC g. ?entilation with high partial
pressures of o'ygen is started* -ut the neonatologist is concerned a-out the possi-le development of retinopathy
of prematurity (formerly called retrolental fi-roplasia). What is the underlying mechanism -y which this retinal
lesion may develop?
A. 4anglion cell degeneration
". 7nappropriate vascular proliferation
. Eptic nerve degeneration
D. 6igment deposition in retina
$. 6igmented epithelium degeneration
$'planation(
)he correct answer is ". )he underlying lesion in retinopathy of prematurity (retrolental fi-roplasia) is an
inappropriate proliferation of vessels in the inner layers of the retina. 7f the process is disrupted early* the retina
may heal with little residual damage. <owever* persistence of the process predisposes for permanent damage
due to e'udation* hemorrhage* and secondary detachment of the retina.
4anglion cell and optic nerve degeneration (choices A and ) are features of retinal damage due to glaucoma.
6igment deposition in the retina (choice D) is unrelated to retinopathy of prematurity.
6igmented epithelium degeneration (choice $) is a feature of retinitis pigmentosa and also of senile macular
degeneration.
A !:-year-old male complains of fatigue and increased fre.uency of urination. Fuestioning reveals that he is
somewhat confused. 6hysical e'amination reveals a :@HC8* 9!C l-. individual whose fat is centrally distri-uted.
Drine is positive for glucose* -ut negative for /etones. A -lood sample drawn from this patient is li/ely to e'hi-it
which of the following compared to that of a normal individual?
A. Decreased concentration of -peptide
". Decreased p<
. 7ncreased anti-odies against islet cell proteins
D. 7ncreased concentration of 2-eta3-hydro'y-utyrate
$. 7ncreased osmolarity
$'planation(
)he correct answer is $. An o-ese adult with glucosuria* -ut not /etonuria* li/ely has type 9 dia-etes mellitus.
)ype 9 dia-etes is characteriAed -y insulin resistance resulting in hyperglycemia and increased serum
osmolarity. )he dehydration associated with osmotic diuresis ma/es the hyperosmolarity worse. As the
osmolarity increases a-ove >>C mEsm,=* the osmotic loss of water from neurons is sufficient to produce coma.
7n non/etotic* hyperosmolar coma* -lood glucose values can range from #CC to 9*!CC mg,d= and produce
serum osmolarities of >>C-!!C mEsm,=.
Since type 9 dia-etes is due to insulin resistance* plasma levels of insulin are usually normal to increased.
"ecause 2-eta3 cells secrete insulin and -peptide in a H(H ratio* plasma concentration of -peptide would also
-e normal to increased (not decreased* choice A) in type 9 dia-etes.
$ven small amounts of insulin are sufficient to prevent /etosis. 7n type 9 dia-etes there is enough insulin effect
to prevent significant lipolysis and su-se.uent formation of e'cess /etone -odies. <ence* acidosis (choice ") is
not typically associated with this disorder. Since 2-eta3-hydro'y-utyrate is a /etone -ody* its concentration in
plasma is not li/ely to -e increased (choice D).
While there is a significant autoimmune component to type H dia-etes* type 9 dia-etes is not associated with
increased circulating anti-odies (choice ) against 2-eta3 cell proteins such as glutamic acid decar-o'ylase.
A I-year-old child is -rought to the pediatrician -ecause of increased thirst and -ed-wetting. Despite an
increased appetite* she has lost : pounds over the past month. A dipstic/ test of a urine sample reveals the
presence of glucose. A -lood sample drawn from this patient is most li/ely to e'hi-it which of the following
compared to that of a normal individual?
A. Decreased concentration of 2-eta3-hydro'y-utyrate
". Decreased concentration of hemoglo-in AHc
. Decreased osmolarity
D. 7ncreased anti-odies against glutamic acid decar-o'ylase (4AD)
$. 7ncreased concentration of -peptide
$'planation(
)he correct answer is D. A child with glucosuria* polyuria* polydipsia* and weight loss despite increased appetite
is li/ely to have type H dia-etes mellitus. )his form of dia-etes has an important autoimmune component to its
etiology. As many as #:; of patients have islet cell autoanti-odies in their -lood in the first few wee/s after
onset of the disease. %ost of these anti-odies are directed against glutamic acid decar-o'ylase* an enAyme that
is present in 2-eta3 cells.
)he insulin deficiency associated with autoimmune destruction of the 2-eta3 cells leads to increased -rea/down
of triglycerides with a conse.uently increased rate of formation of /etone -odies. )he primary /etone -ody
produced is 2-eta3-hydro'y-utyrate (choice A)* the -lood level of which is increased (not decreased) in type H
dia-etes. 7f untreated* type H dia-etes can rapidly progress to /etoacidosis and coma.
7nsulin deficiency leads to decreased glucose upta/e and hyperglycemia. )he increase in -lood glucose results
in increased glycosylation of hemoglo-in in red -lood cells. <ence* the hemoglo-in AHc concentration will -e
increased* not decreased (choice ")* reflecting long-standing hyperglycemia.
Another conse.uence of the increased -lood glucose is increased serum osmolarity (not decreased* choice ).
7f the osmolarity e'ceeds >>C mEsm,=* consciousness -egins to -e impaired.
2-eta3 cell destruction in type H dia-etes leads to decreased secretion of insulin. Since insulin and -peptide
are released on a H(H ratio -y the 2-eta3 cells* the secretion of -peptide is also decreased (not increased*
choice $). 7nsulin is first synthesiAed as preproinsulin* with an initial leader se.uence of hydropho-ic amino acids
that allows the developing protein to -e e'truded from the ri-osomes into the rough endoplasmic reticulum. )he
8pre8 segment is then cleaved off* leaving proinsulin. 6roinsulin is pac/aged into secretory granules within the
4olgi* and as the granules -ud off* enAymes cleave the precursor into insulin and -peptide.
A KI-year-old man presents to his doctor@s office for an insurance physical. During -lood pressure measurement*
the nurse notes that systolic sounds are heard with the cuff completely deflated. )he -lood pressure is H#C,KC
mm <g. 6hysical e'amination reveals -ounding pulses and a high-pitched* -lowing diastolic murmur* heard -est
along the left sternal -order. Which of the following is the most li/ely diagnosis?
A. Aortic regurgitation
". Aortic valve o-struction
. ardiac tamponade
D. <eart failure
$. <ypovolemia
$'planation(
)he correct answer is A. )he case descri-ed is typical for aortic regurgitation. 7n aortic regurgitation* -lood
flows -ac/ward through the aortic valve during diastole when the valve is closed. )he arterial pulse pressure is
widened* often to over HCC mm <g (normal pulse pressure is >C to :C mm <g)* and in e'treme cases the
systolic pressure can -e elevated to over 9:C mm <g with a depression in the diastolic pressure. &ote that the
pulse pressure is H#C-KC O H9C mm <g in this case. )he aortic pressure falls greatly during diastole -ecause
-lood from the aorta regurgitates -ac/ward into the ventricle through the lea/y aortic valve. )he increase in
systolic pressure most li/ely results from the large increase in stro/e volume* which is secondary to an increase
in the end diastolic volume. )he diastolic murmur of aortic regurgitation is typically a high-pitched* -lowing*
decrescendo murmur* -est heard along the left sternal -order over the third intercostal space.
)he pulse pressure is decreased in aortic valve o-struction (choice ")* cardiac tamponade (choice )* heart
failure (choice D)* and hypovolemia (choice $).
A !K-year-old woman presents to her doctor complaining of wea/ness and fatigue. En physical e'amination* her
physician notices a HC-pound weight gain since her last visit K months ago. <er -lood pressure is HKC,HCC mm
<g. "lood tests reveal serum &a1 H:: m$.,=* 51 9.# m$.,=* and a decreased serum renin. Which of the following
is the most li/ely diagnosis?
A. ushing@s syndrome
". Dia-etes mellitus
. 6heochromocytoma
D. 6rimary aldosteronism
$. Secondary aldosteronism
$'planation(
)he correct answer is D. 6rimary aldosteronism (onn@s syndrome) is a condition of hyperaldosteronism
originating in the adrenal gland. )he causes include an aldosterone-secreting adrenocortical adenoma*
hyperplasia of the Aona glomerulosa* and very rarely* an adrenal carcinoma. 7t is characteriAed -y hypertension
secondary to sodium retention* hypo/alemia* and a decreased serum renin due to a negative feed-ac/ of
increased -lood pressure on renin secretion.
ushing@s syndrome (choice A) is the result of increased glucocorticoid production* particularly cortisol. 6hysical
signs typically include 8moon facies*8 truncal o-esity* 8-uffalo hump*8 and purple a-dominal striae.
Dia-etes mellitus (choice ") is a condition of inade.uate insulin production that presents with hyperglycemia
and /etoacidosis.
6heochromocytoma (choice ) is a rare tumor of chromaffin cells occurring most commonly in the adrenal
medulla. )he tumor secretes epinephrine and norepinephrine* resulting in secondary hypertension.
Secondary aldosteronism (choice $) results from an activation of the renin-angiotensin system caused -y renal
ischemia* edema* and renal tumors. 7n contrast to primary aldosteronism* secondary aldosteronism is
associated with increased serum renin.
A K:-year-old man with hyperlucent lung fields develops e'treme shortness of -reath over a period of a-out H:
minutes. hest '-ray shows a shift of the mediastinum to the right* and the lung field on the left appears even
more hyperlucent than -efore* with the e'ception of a white shadow near the heart -order. Which of the following
is the most pro-a-le cause of the patient@s current pro-lem?
A. "ronchogenic carcinoma
". 6leural effusion
. 6ulmonary em-olism
D. Rupture of an emphysematous -ulla
$. )u-erculosis
$'planation(
)he correct answer is D. )he patient@s initial hyperlucent lung fields strongly suggest the presence of
emphysema. )he radiologic findings after the increase in shortness of -reath are consistent with free air in the
chest* which has collapsed the left lung and caused a shift in the location of the mediastinum. Such air might
have -een introduced -y rupture of a -ulla. Small pneumothoraces are usually well tolerated* -ut larger ones
may re.uire decompression (the needle from a syringe is sometimes used)* or even surgical correction if -ullae
continue to lea/ air.
"ronchogenic carcinoma (choice A) would -e e'pected to produce a mass lesion.
6leural effusion (choice ") usually develops slowly* and causes a whitening of lung fields when fluid is present.
6ulmonary em-olism (choice ) can cause a sudden shortness of -reath* -ut would not cause an increase in
the lucency of the lung fields.
)u-erculosis (choice $) would -e e'pected to produce a mass lesion in the lung.
A 9-year-old infant with uncomplicated coarctation of the aorta appears to -e in good health. 4rowth and
development are normal. )he constriction is located +ust distal to the su-clavian arteries. Which of the following
is decreased in this patient?
A. "lood flow in the upper -ody
". "lood flow in the lower -ody
. "lood pressure in the upper -ody
D. ?ascular resistance in the upper -ody
$. ?ascular resistance in the lower -ody
$'planation(
)he correct answer is $. 7n fully compensated aortic coarctation* -lood flow is normal in the upper and lower
-ody (choices A and ") despite an increased arterial pressure (a-out :C; higher) in the upper -ody (choice
) compared to the pressure in the lower -ody. "ecause resistance O pressure,-lood flow* it is clear that
resistance must -e lower in the lower portions of the -ody. )he mechanism of this decrease in resistance -elow
the constriction (and increased resistance a-ove the constriction) is autoregulation of -lood flow. )he small
arteries and arterioles dilate (or constrict) in accordance with the meta-olic needs of the tissues ensuring that
each tissue receive an ade.uate amount of -lood flow. )hus* the increase in -lood pressure in the upper -ody
leads to constriction of the arterioles* which increases vascular resistance (choice D)* and the lower pressure
-elow the coarctation leads to dilation of the arterioles* which decreases vascular resistance in the lower -ody.
7n which of the following conditions would o'ygen therapy -e most effective in alleviating hypo'ia?
A. Anemia due to -lood loss
". $dematous tissues
. $mphysema
D. =ocaliAed circulatory deficiencies
$. Right-to-left cardiac shunts
$'planation(
)he correct answer is . hronic pulmonary emphysema is characteriAed -y distention of small air spaces
distal to the respiratory -ronchioles and destruction of alveolar septa. =ong-term cigarette smo/ing is the usual
cause. )he mar/ed loss of lung parenchyma associated with emphysema leads to a decrease in the diffusion
capacity of the lungs* which reduces their a-ility to o'ygenate -lood and remove car-on dio'ide. When arterial
hypo'emia is persistent and severe* o'ygen therapy should therefore -e considered.
E'ygen therapy is less effective for treating the hypo'ia associated with anemia (choice A)* edematous tissues
(choice ")* localiAed circulatory deficiencies (choice D)* and right-to-left cardiac shunts (choice $)* -ecause in
each case* there is already ade.uate o'ygen availa-le in the alveoli. )he pro-lem in each of these situations is
inade.uate transport of o'ygen to the tissues* -lunting the effects of increasing the o'ygen tension of the
inspired air. E'ygen therapy will nonetheless increase the amount of dissolved o'ygen carried in the -lood*
which may -e life-saving in some instances.
A middle-aged woman comes to her physician@s office with complaints of visual difficulties. A review of systems
and physical e'amination are unremar/a-le e'cept for her eye e'am. When a light is shined in her right eye*
there is no pupillary response in either eye. <owever* upon shining a light in her left eye* -oth ipsilateral and
contralateral pupillary responses are apparent. <er e'traocular movements are intact. What is the most li/ely
location of her lesion?
A. Eculomotor nerve* left side
". Eculomotor nerve* right side
. Eptic nerve* left side
D. Eptic nerve* right side
$. )rochlear nerve* left side
0. )rochlear nerve* right side
$'planation(
)he correct answer is D. 5now your cranial nervesQ )his woman has a 8%arcus-4unn pupil*8 with a defect in the
afferent pathway of the optic nerve (in this case on the right side). Recall that the afferent lim- of the pupillary
light refle' is the optic nerve (& 77)3 the efferent lim- is the oculomotor nerve (& 7773 parasympathetic fi-ers).
When light is shined into her right eye* -ecause her right optic nerve is not functioning properly* the light signal
is not transmitted to the &S* resulting in no pupillary response. As light is shined into her left eye* the left optic
nerve transmits the signal to the &S* which then sends an out-ound signal through -oth the right and left
oculomotor nerves to cause pupillary constriction in -oth eyes.
)he oculomotor nerve (choices A and ") innervates all e'traocular muscles e'cept the lateral rectus
(innervated -y the a-ducens nerve)* and the superior o-li.ue (innervated -y the trochlear nerve3 choices $
and 0). )he oculomotor nerve also mediates pupillary constriction (parasympathetic fi-ers)* eyelid opening
(levator palpe-rae)* and innervates the ciliary muscle (allowing accommodation).
A patient with a lesion of the left optic nerve (choice ) would have no pupillary responses in either eye when
shining a light in the left eye3 pupillary responses would -e present in -oth eyes when shining a light in the right
eye.
En a routine physical e'amination for medical insurance* a midsystolic e+ection murmur is detected in the
pulmonic area of a >:-year-old female e'ecutive. )he cardiac e'amination also revealed a prominent right
ventricular cardiac impulse and wide and fi'ed splitting of the second heart sound. An $54 showed a right a'is
deviation and chest '-ray showed enlargement of the right ventricle and atrium. Which of the following is the most
li/ely diagnosis?
A. Aortic stenosis
". Atrial septal defect
. %itral regurgitation
D. %itral stenosis
$. 6ulmonary valve stenosis
$'planation(
)he correct choice is ". )he classic findings in atrial septal defect are a prominent right ventricular cardiac
impulse* a systolic e+ection murmur heard in the pulmonic area and along the left sternal -order* and fi'ed
splitting of the second heart sound. )hese findings are due to an a-normal left to right shunt through the defect*
creating a volume overload on the right side. )he increase in volume on the right side creates the flow murmur*
the dilatation of the right-sided cham-ers* and the delayed closure of the pulmonic valve* all of which are
present in this case.
Aortic stenosis (choice A) is also associated with a systolic e+ection murmur. )he murmur is usually loudest at
the right sternal -order and radiates upward to the +ugular notch. )his condition is associated with left
ventricular hypertrophy.
%itral regurgitation (choice ) would present with a systolic murmur as well. <owever* left atrial enlargement
would -e seen -efore right ventricular enlargement.
%itral stenosis (choice D) would present with an 8opening snap8 and a diastolic murmur.
6ulmonary valve stenosis (choice $) causes an increase in right ventricular pressure resulting in right
ventricular hypertrophy and pulmonary artery dilatation. A crescendo-decrescendo murmur may -e heard if the
stenosis is severe. Right atrial enlargement would not -e present.
During a -o'ing match* a contestant is 8/noc/ed out8 -y a -low to the lateral s/ull. <e recovers after a few
minutes* and is asymptomatic for the ne't H9 hours. <e then develops a severe headache* changes in mental
status* nausea* and vomiting. Which of the following is the most li/ely diagnosis?
A. "asilar s/ull fracture
". $pidural hemorrhage
. 7ntracere-ral hemorrhage
D. Su-arachnoid hemorrhage
$. Su-dural hematoma
$'planation(
)he correct answer is ". All of the lesions listed in the answers can occur in -rain trauma. <owever* the
scenario descri-ed is classic for epidural hemorrhage. 7n this scenario* a severe -low to the lateral s/ull causes
-oth s/ull fracture and laceration of the middle meningeal artery (/now the name of this artery* as it is
fre.uently tested)* leading to a momentary loss of consciousness* which is followed -y a lucid (asymptomatic)
period of H-!# hours -efore the patient@s neurologic condition deteriorates. )his scenario is fre.uently tested in
e'aminations* -ut you should -e aware that in real clinical life* there may -e no initial loss of consciousness and
the patient may not have sought medical help for s/ull trauma.
"asilar s/ull fractures (choice A) are usually located in the vicinity of the petrous -one or along the sphenoid
-one. linical signs of -asilar s/ull fracture include hemotympanum (-lood visi-le -ehind the tympanic
mem-rane)* delayed ecchymosis over the mastoid process ("attle@s sign)* or perior-ital ecchymosis (8raccoon
sign8). S0 lea/age (sometimes from the nose)* or pneumocephalus may also occur.
7ntracere-ral hemorrhage (choice ) is most often due to hypertension* anticoagulant use* cere-ral amyloid
angiopathy* or cocaine and,or methamphetamine a-use.
Su-arachnoid hemorrhage (choice D) is most commonly caused -y rupture of a cere-ral -erry aneurysm* -ut
may also -e associated with arteriovenous malformations or intraparenchymal hemorrhage.
Su-dural hematomas (choice $) are due to rupture of the -ridging veins -etween the periosteal dura and the
superficial cere-ral veins. "lood accumulates -elow the dura* producing symptoms of increased intracranial
pressure in minutes to hours (acute su-dural hematomas)* or after wee/s or months (chronic su-durals* often
seen in the elderly). An acute su-dural may present li/e an epidural hemorrhage* -ut a forceful -low to the
lateral s/ull producing s/ull fracture is generally associated with an epidural* rather than a su-dural* -leed.
A II-year-old -lac/ male is recovering in a reha-ilitation center HC days after having sustained a massive
anterolateral infarction of the left ventricle. <e suddenly develops paralysis of the entire right half of the -ody*
including facial and eye muscles. Which of the following complications of myocardial infarction is most li/ely to
have precipitated this event?
A. alcific coronary atherosclerosis
". $lectromechanical dissociation
. =eft -undle -ranch -loc/
D. %yocardial rupture
$. ?entricular mural throm-us
$'planation(
)he correct answer is $. )his patient has suffered a cere-rovascular accident as a conse.uence of
throm-oem-oli emanating from a mural throm-us formed over the recent myocardial infarction. %ural throm-us
fre.uently develops over a previously infarcted segment of myocardium* especially when the infarction is large
and a ventricular aneurysm develops. 7schemic damage to the endocardium* solu-le factors released -y the
in+ured myocardium* and altered wall /inetics that produce sluggish -lood flow all favor mural throm-us
formation. Ene important conse.uence of a mural throm-us is throm-oem-olism to the systemic circulation*
producing a stro/e* as in this patient@s case.
alcific coronary atherosclerosis (choice A) is very li/ely responsi-le for the patient@s original myocardial
infarction. oronary atherosclerosis is not a conse.uence of myocardial infarction* and it does not directly
predispose to the development of em-olic stro/es. omplicated atherosclerotic lesions in the carotid or cere-ral
circulation* however* may directly lead to a cere-rovascular accident.
$lectromechanical dissociation ($%D3 choice ") is a catastrophic event that fre.uently leads to sudden death*
not stro/e. 7n $%D* although a normal potential is transmitted through the myocardium* no pulse of arterial
-lood is sent to the systemic circulation. $%D may occur as a conse.uence of pericardial tamponade* massive
pulmonary em-olism* or myocardial to'ins that prevent normal cardiac muscle contraction.
=eft -undle -ranch -loc/ (choice ) represents failure of electrical transmission along the 6ur/in+e fi-ers to the
left ventricular myocardium. "undle -ranch -loc/s are important causes of arrhythmia* -ut are not responsi-le
for cere-rovascular accidents.
%yocardial rupture (choice D) is an infre.uent conse.uence of myocardial infarction that typically occurs
appro'imately H wee/ after the infarction. 7f scarring of the infarcted segment does not /eep pace with necrosis
and digestion of the infarcted tissue* the myocardium can rupture under the high intraventricular pressures* and
pericardial tamponade generally ensues. %yocardial rupture rapidly leads to acute heart failure* not stro/e.
6ulmonary artery pressure(
99,I mm <g
6ulmonary wedge pressure(
HC mm <g
=eft ventricular pressure(
HIC,: mm <g
Aortic pressure(
H9C,#C mm <g
A :C-year-old male complaining of chest pain is admitted to the emergency room. <e is ta/en to the cardiac
catheteriAation la-oratory on the same day. Selected pressures are shown a-ove. What is the most li/ely
diagnosis?
A. Aortic regurgitation
". Aortic stenosis
. %itral regurgitation
D. %itral stenosis
$'planation(
)he correct answer is ". )his patient has aortic stenosis. 7n aortic stenosis* the -lood is e+ected from the left
ventricle through a smaller-than-normal opening. "ecause the resistance to e+ection of -lood is high* the left
ventricular pressure can sometimes increase to over 9:C mm <g with normal systolic pressure in the aorta.
&ote that in this patient* the left ventricular pea/ systolic pressure has increased to HIC mm <g* the aortic
systolic pressure is normal at H9C mm <g* and a :C mm <g gradient e'ists across the aortic valve during
systole.
7n aortic regurgitation (choice A)* -lood flows -ac/ward through the aortic valve during diastole when the valve
is closed. =eft ventricular systolic pressure and aortic systolic pressure are nearly the same with pure aortic
regurgitation.
%itral regurgitation (choice ) means there is -ac/ward flow of -lood through the mitral valve during systole.
)his accumulation of e'tra -lood in the left atrium during ventricular systole leads to an elevation in the
pulmonary wedge pressure (which is used as an estimate of left atrial pressure).
7n mitral stenosis (choice D)* -lood must flow from the left atrium into the left ventricle through a
smaller-than-normal opening. "ecause the resistance to -lood flow through the mitral valve is higher than
normal* the left atrial pressure (estimated -y the pulmonary wedge pressure) is often several mm <g higher
than the left ventricular end diastolic pressure.
A K>-year-old man with essential hypertension has gone several wee/s without ta/ing his medications. <e arrives
at the emergency room with a severe laceration on his right hand after falling on a -ro/en -eer -ottle. <e has a
heart rate of JC -eats per minute and a -lood pressure of HIC,HH:. Which of the following is most li/ely to -e
decreased in the s/eletal muscles of his legs?
A. Adenosine levels
". Arterial -lood pressure
. Arteriolar resistance
D. "lood flow
$. ?enous o'ygen concentration
$'planation(
)he correct answer is A. )he s/eletal muscles of the -ody have a normal -lood flow even when -lood pressure
is chronically elevated. Ergans and tissues in which the vasculature has primarily a nutritive function (e.g.*
-rain* heart* and s/eletal muscle) regulate their -lood flow in accordance with the meta-olic needs of the
tissues. )hese tissues e'hi-it short-term autoregulation of -lood flow such that the increase in flow caused -y
an elevated arterial pressure is minimiAed -y constriction of the arterioles. )he constriction is caused in part -y
decreased levels of adenosine (an endogenous vasodilator) in the tissues. )he rate of adenosine production in
a tissue is a function of its meta-olic rate* which is not affected significantly -y an increase in systemic
pressure. When -lood flow to the muscle increases* the adenosine is literally washed from the muscle* lowering
the tissue levels of adenosine. )he decrease in adenosine concentration causes small arteries and arterioles in
the muscle to constrict and this increase in resistance (choice ) maintains -lood flow (choice D) at a normal
rate in the face of increased arterial pressure (choice "). )he overall process is called autoregulation of -lood
flow.
?enous o'ygen concentration (choice $) does not decrease in the s/eletal muscles of hypertensives -ecause
-lood flow is maintained at an ade.uate level to meet the nutritional demands of the muscles.
Which of the following conditions would mostly li/ely -e associated with chronic gastritis ()ype A) resulting from
autoimmune destruction of parietal cells?
A. Decreased growth of luminal -acteria
". Decreased li/elihood of developing gastric carcinoma
. Decreased plasma concentration of gastrin
D. 7ncreased production of macrocytic red -lood cells
$. 7ncreased secretion of pancreatic -icar-onate
$'planation(
)he correct answer is D. Autoimmune destruction of parietal cells would lead to decreased secretion of gastric
acid and intrinsic factor. )he diminished availa-ility of intrinsic factor would result in poor a-sorption of dietary
vitamin "H9. Ever time* the vitamin "H9 deficiency could lead to pernicious anemia* which is characteriAed -y
increased production of macrocytes (megalo-lasts) -y the -one marrow.
"ecause of the decrease in gastric acid secretion* luminal -acteria (choice A) would most li/ely e'hi-it
increased (not decreased) growth. Ene of the functions of <l secreted -y the parietal cells is to steriliAe the
gastric lumen.
6atients with )ype A gastritis have an increased li/elihood of developing gastric carcinoma (not decreased*
choice ").
A decrease in acid secretion leads to increased secretion of gastrin (not decreased* choice ) -y antral 4
cells. )his is -ecause low gastric p< (less than >) inhi-its gastrin secretion via paracrine release of
somatostatin from cells in the gastric mucosa that can sense the acidity. With decreased parietal cells* the p<
of the gastric lumen would rise and remove this inhi-itory component.
"ecause less acid would -e delivered to the duodenum with parietal cell destruction* less secretin would -e
released into the -lood.. )his would result in decreased pancreatic -icar-onate secretion (not increased*
choice $).
A ::-year-old hypertensive man develops sudden onset of e'cruciating pain -eginning in the anterior chest* and
then radiating to the -ac/. Ever the ne't 9 hours* the pain moves downward toward the a-domen. Which of the
following is the most pro-a-le diagnosis?
A. Aortic dissection
". Aortic valve stenosis
. Atherosclerotic aneurysm
D. %yocardial infarction
$. Syphilitic aneurysm
$'planation(
)he correct answer is A. )his patient has an aortic dissection (formerly called dissecting aneurysm)* a
potentially fatal condition that is too often confused clinically with myocardial infarction. )he most important
clinical clue is that the pain shifts with time. &on-invasive techni.ues such as transesophageal
echocardiography* computed tomography ())* and magnetic resonance imaging (%R7) are increasingly useful
in ma/ing this diagnosis.
Aortic valve stenosis (choice ") would not -e e'pected to produce severe chest pain of acute onset.
)his patient@s clinical history does not suggest either an atherosclerotic (choice ) or a syphilitic (choice $)
aneurysm. $ven if he had one of either of these types of aneurysms and it had -egun to rupture* the distinctive
feature of severe pain moving downward would pro-a-ly not -e present.
%yocardial infarction (choice D) is the ma+or diagnosis most often confused with this patient@s condition. )he
movement of the pain is the ma+or clinical tip-off suggesting that this is not the correct answer.
A neurologist gives a car accident victim a neurological e'amination. As part of the e'amination* he presents the
patient with a picture of a dog and as/s her to tal/ a-out it. She seems una-le to name or recogniAe the picture.
<e then as/s her to copy a picture of the dog* which she is a-le to do* although she still is una-le to identify the
animal as a dog. Which of the following is the most appropriate diagnosis?
A. Agnosia
". Ale'ia
. Aphasia
D. Apra'ia
$. Dysle'ia
$'planation(
)he correct answer is A. Agnosia is the ina-ility to recogniAe* despite ade.uate sensation. )his patient has a
type of agnosia /nown as visual agnosia* which is the ina-ility to recogniAe familiar o-+ects despite the a-ility to
see. )he fact that she could copy a picture of the dog indicates that she has ade.uate visual acuity* visual
fields* and perception. 7f this patient had only visual agnosia* if she were to hear a dog -ar/ (auditory) or pet a
dog (tactile)* then she would -e a-le to recogniAe and name the dog. 6rosopagnosia is a special type of visual
agnosia in which a person is una-le to recogniAe faces. )here are also other types of agnosias* including
auditory and tactile agnosias.
Ale'ia (choice ") is an ac.uired reading disorder due to -rain damage. 6eople with this disorder can
comprehend words that are spelled out loud* and can understand words and letters that are written on the
palm* -ut they cannot read.
Aphasia (choice ) is an ac.uired disorder of language due to -rain damage. $'amples are "roca@s aphasia
and Wernic/e@s aphasia.
Apra'ia (choice D) is the loss of the a-ility to carry out certain movements correctly in response to stimuli that
normally elicit them. )his occurs even though the patient has no sensory loss* no wea/ness* and no
distur-ance of language comprehension.
Dysle'ia (choice $) is a developmental reading disorder.
A K9-year-old man is admitted to the hospital -ecause of symptoms of congestive heart failure* which have -een
present for the past few years. 0ifteen years -efore admission* the man sustained a /nife wound to the left
supraclavicular area. 6hysical e'amination reveals signs of a large* left su-clavian arteriovenous fistula. Which
of the following is e'pected to -e present in this man?
A. Decreased heart rate
". Decreased stro/e volume
. Decreased mi'ed venous o'ygen content
D. 7ncreased resting cardiac output
$. 7ncreased diastolic -lood pressure
0. 7ncreased systemic vascular resistance
$'planation(
)he correct answer is D. 7ncreased cardiac output can -e demonstrated when there is a large fistula that
involves a ma+or artery such as the aorta* su-clavian artery* femoral artery* common carotid artery* or iliac
artery. )he increase in cardiac output caused -y the fistula is roughly e.ual to the -lood flow through the
fistula.
)he increase in cardiac output is associated with increases in -oth heart rate (choice A) and stro/e volume
(choice ").
)he mi'ed venous o'ygen content (choice ) is increased -ecause o'ygenated arterial -lood flows through the
fistula into the venous system.
)he diastolic -lood pressure (choice $) falls -ecause -lood can rapidly e'it the arterial system through the
fistula* -ut mean -lood pressure is maintained relatively constant secondary to increased -lood volume caused
-y renal retention of salt and water.
"ecause -lood is shunted from an artery to a vein through a low resistance pathway* the systemic vascular
resistance (choice 0) is decreased.
A couple presents to a clinic for wor/-up of infertility after : years of unprotected intercourse. )he wife denies
any medical pro-lems and notes regular menstrual cycles. )he hus-and states that he has had chronic sinusitis
and lower respiratory tract infections. 6hysical e'amination of the woman is unremar/a-le. $'amination of the
man is remar/a-le for de'trocardia. 0urther wor/-up of the hus-and will most li/ely reveal
A. aAoospermia
". germinal cell aplasia
. immotile sperm
D. isolated gonadotropin deficiency
$. varicocele
$'planation(
)he correct answer is . )he hus-and is suffering from 5artagener@s syndrome* an autosomal recessive
disorder characteriAed -y infertility* situs inversus* chronic sinusitis* and -ronchiectasis. )he underlying cause
of these varied manifestations are defects in the dynein arms* spo/es of microtu-ule dou-lets of cilia in the
airways and the reproductive tract. Since sperm motility is dependent on the functioning of cilia* infertility
fre.uently accompanies this disorder. Situs inversus occurs -ecause ciliary function is necessary for cell
migration during em-ryonic development.
AAoospermia (choice A) is not a feature of 5artagener@s syndrome* as sperm production or survival is not
affected in this disorder.
4erminal cell aplasia (choice ")* also /nown as Sertoli only syndrome* is characteriAed -y oligospermia or
aAoospermia.
7solated gonadotropin deficiency (choice D) is characteriAed -y delayed or incomplete pu-ertal maturation.
?aricocele (choice $) results in an increased testicular temperature* decreasing the count of normal* via-le
sperm.
A patient with long-standing constipation enters a clinical research study. After a complete physical e'amination*
a small intraluminal -alloon is inserted through the anus to the rectum. )ransducers are also inserted to measure
internal and e'ternal anal sphincter pressures. 7nflation of the rectal -alloon causes the e'ternal anal sphincter to
contract* -ut the internal anal sphincter* which e'hi-its normal tone* fails to rela' and the urge to defecate is not
sensed. Which of the following structures is most li/ely damaged?
A. 7nternal anal sphincter
". $'ternal anal sphincter
. 6elvic nerve
D. 6udendal nerve
$. Rectum
$'planation(
)he correct answer is . )he defecation refle' that is evo/ed when the rectum is distended involves three
responses( H) the internal anal sphincter rela'es* 9) the e'ternal anal sphincter contracts* and >) a conscious
urge to defecate is perceived. )his neural refle' involves the pelvic nerve* which provides the parasympathetic
preganglionic innervation to the internal anal sphincter (composed of smooth muscle) and also carries the
sensory afferent information from the rectum to the spinal cord* and the pudendal nerve* which carries the
somatic efferent inputs to the e'ternal anal sphincter (composed of s/eletal muscle). 7n the patient descri-ed
a-ove* the pelvic nerve is most li/ely damaged since neither the refle' rela'ation of the internal anal sphincter
nor the urge to defecate is evo/ed -y rectal distention.
Damage to the internal anal sphincter (choice A) would most li/ely cause resting tone to -e low and* if anything*
lead to fecal incontinence rather than constipation. 0urthermore* damage to the internal anal sphincter could
not e'plain the failure of the appearance of the urge to defecate.
)he e'ternal anal sphincter (choice ") appears to function normally since distention of the rectum evo/es the
e'pected contraction. )he normal contraction of the e'ternal anal sphincter also suggests that the pudendal
nerve (choice D) is intact.
Since distention of the rectum (choice $) evo/ed the contraction of the e'ternal anal sphincter* it too appears to
-e functioning normally.
A KC-year-old man complains of difficulty arising from a chair and initiating new movements. En e'am* you notice
a resting hand tremor and cogwheel rigidity. Which of the following amino acids is the precursor for the
neurotransmitter that is deficient in the -rain of this patient?
A. 4lutamate
". 4lycine
. <istidine
D. )ryptophan
$. )yrosine
$'planation(
)he correct answer is choice $. )his .uestion re.uires three steps of logic. 0irst* figure out the diagnosis (a
classic case of 6ar/inson@s disease)* then remem-er which neurotransmitter is involved in the disease
(dopamine)* and finally* recall which amino acid serves as the precursor for that neurotransmitter (tyrosine).
)he hydro'ylation of tyrosine -y tyrosine hydro'ylase results in DE6A* which is then decar-o'ylated to
dopamine. &ote that norepinephrine and epinephrine are also tyrosine derivatives* as are the melanins and the
thyroid hormones* thyro'ine and triiodothyronine.
)he other answer choices are also precursors of specialiAed products(
4lutamate (choice A) can -e converted to the inhi-itory neurotransmitter 4A"A -y the action of glutamate
decar-o'ylase.
4lycine (choice ") is involved in the synthesis of -oth creatine (along with arginine and S-adenosylmethionine)
and heme (along with succinyl-oA).
<istidine (choice ) can -e decar-o'ylated to histamine* an important inflammatory mediator.
)ryptophan (choice D) can -e converted to serotonin -y a hydro'ylation reaction (tryptophan hydro'ylase)
followed -y a decar-o'ylation reaction.
A !C-year-old woman presents to the emergency department -ecause of hematuria. =a-oratory analyses show
significant proteinuria* -acteria and white cells in the urine* and a -lood urea nitrogen ("D&) of !C mg,d= with a
creatinine of !.C mg,d=. Dltrasonography reveals enlarged /idneys* and she is given a provisional diagnosis of
polycystic renal disease. Which of the following is li/ely to -e decreased in this patient?
A. reatinine clearance
". $'tracellular sodium concentration
. 4lucose clearance
D. 6lasma creatinine levels
$. 6lasma inulin levels
$'planation(
)he correct answer is A. 6olycystic /idney disease leads to progressive decrements in renal function*
eventually resulting in renal failure (evidenced -y the increased "D& and creatinine). )hese a-normalities are
caused -y a reduction in glomerular filtration rate (40R)* which produces a decrease in creatinine clearance.
)he production of creatinine* a waste product of meta-olism* -ears a direct relation to the muscle mass of an
individual and is independent of renal function. "ecause creatinine is freely filtered -y the glomerulus* -ut not
secreted or rea-sor-ed to a significant e'tent* the renal clearance of creatinine is appro'imately e.ual to the
40R. )herefore* creatinine clearance is commonly used to assess renal function in the clinical setting.
)he e'tracellular sodium concentration (choice ") is not e'pected to change significantly in this patient.
4lucose (choice ) is not normally e'creted* so glucose clearance is normally Aero. )hus* is not possi-le for
glucose clearance to -e decreased.
6lasma creatinine concentration (choice D) increases when 40R decreases.
7nulin clearance is used to estimate 40R. <owever* inulin is foreign to the -ody* and inulin plasma levels
(choice $) are normally Aero.
)wo dia-etic patients are seen -y a clinician. )he first patient is a HK-year-old -oy who 9 years previously had
presented with polyuria and polydipsia. )he second patient is a K:-year-old woman whose dia-etes was
identified -y the presence of hyperglycemia on a routine -lood glucose screen HC years previously. ompared to
the K:-year-old patient with dia-etes* the HK-year-old dia-etic is more li/ely to
A. -e o-ese
". -ecome euglycemic with oral hypoglycemic agents
. develop /etoacidosis
D. have relatively high endogenous insulin levels
$. not have the <=A-DR> or <=A-DR! allele
$'planation(
)he correct answer is . )he HK-year-old pro-a-ly has type H (+uvenile onset) dia-etes mellitus* while the
K:-year-old pro-a-ly has type 9 (maturity onset) dia-etes mellitus. )hese two types of dia-etes differ in many
respects. 5etoacidosis is more apt to develop in type H dia-etes.
type 9 dia-etics tend to -e o-ese (choice A)* while type H dia-etics are often thin.
type H is usually apparently due to viral or immune destruction of -eta cells* while type 9 is apparently usually
due to increased resistance to insulin3 conse.uently the K:-year-old* rather than the HK-year-old* is more li/ely
to have relatively high endogenous levels of insulin (choice ").
type 9 dia-etes can often -e controlled with oral hypoglycemic agents (choice )* while type H dia-etics
generally re.uire insulin. &ote that some type 9 dia-etics also may re.uire insulin as the disease evolves.
type H dia-etes has a strong association with <=A-DR> and <=A-DR! (choice $)* while type 9 does not have
any strong <=A associations.
A KJ-year-old man involved in a motorcycle accident loses an estimated H = of -lood from a severed artery in his
leg. onstriction of which of the following types of -lood vessels is most important for minimiAing the decrease in
mean systemic filling pressure caused -y this -lood loss?
A. Aorta and large arteries
". Arterioles
. apillaries
D. Small arteries
$. ?enules and veins
$'planation(
)he correct answer is $. )he venous system serves as an important -lood reservoir for the circulation. When
-lood is lost from the -ody and -lood pressure -egins to fall* pressure refle'es are elicited that send
sympathetic nerve signals to venules and veins causing them to constrict. "y 8tightening8 the circulation and
ta/ing up much of the 8slac/8 caused -y the -lood loss* nearly normal function can usually -e restored with up
to a 9C; loss of -lood volume.
)he mean systemic filling pressure (%S06) is the pressure that e'ists in all parts of the circulation when the
heart has -een stopped and the -lood volume has -ecome redistri-uted in the system until all pressures are at
e.uili-rium. )he %S06 is thus a measure of the 8tightness8 with which the circulatory system is filled with -lood.
)he more the system is filled (i.e.* when %S06 increases)* the easier it is for -lood to flow into the heart* which
tends to increase venous return.
onstriction of the arterial system has relatively little effect on mean systemic filling pressure -ecause the
arterial system (choices A* "* and D) contains a relatively small volume of -lood.
apillaries do not constrict -ecause they do not contain smooth muscle cells in their walls (choice ).
A 9C-year-old male is evaluated for persistent gynecomastia. 6hysical e'amination reveals the presence of
a-normally small* firm testes. )esticular -iopsy shows fi-rosis and hyaliniAation of the seminiferous tu-ules. )he
=eydig cells are present in clumps and are hyperplastic. hromosomal analysis shows the presence of an GGP
genotype. Which of the following la-oratory findings would -e most li/ely in this individual?
A. Decreased plasma estrogen
". Decreased plasma follicle stimulating hormone
. Decreased plasma luteiniAing hormone
D. Decreased plasma testosterone
$. 7ncreased plasma inhi-in
$'planation(
)he correct answer is D. 5linefelter@s syndrome is characteriAed -y an GGP genotype and is associated with
seminiferous tu-ule dysgenesis. 7n this disorder* there are usually few symptoms -efore pu-erty* which may -e
delayed. At pu-erty* the seminiferous tu-ules fail to enlarge normally and instead undergo fi-rosis and
hyaliniAation. )he result is an ina-ility to produce sperm. )he =eydig cells are hyperplastic and clumped
together* and do not function normally (secretion of testosterone is decreased). As a conse.uence* plasma
levels of =< are increased (not decreased* choice ) due to the loss of feed-ac/ inhi-ition. )he increased =<
stimulates the =eydig cells to increase estrogen (not decreased* choice A) secretion. )he increased
estrogen(testosterone ratio is responsi-le for the gynecomastia* small penis* sparse -ody hair* and other
feminiAed features of 5linefelter@s syndrome. "ecause of dysgenesis of the seminiferous tu-ules* the secretion
of inhi-in is reduced (not increased* choice $). =ow plasma levels of inhi-in result in increased plasma 0S< (not
decreased* choice ") due to loss of feed-ac/ inhi-ition at the pituitary.
.
)he pressure tracings from the thoracic and a-dominal aorta shown a-ove were o-tained from a >-month-old infant
who e'hi-ited dyspnea* difficulty feeding* and poor weight gain. 6hysical e'amination reveals a wea/ femoral pulse
compared to the radial pulse. Which of the following is li/ely to -e higher than normal in this infant?
A. 0emoral artery wall thic/ness
". A-dominal aorta wall thic/ness
. =eft ventricular wall thic/ness
D. Resting -lood flow in leg muscles
$. Renal -lood flow
$'planation(
)he correct answer is . )he infant has a coarctation of the aorta. 7n this condition* the heart must pump against a
higher-than-normal pressure. )he left ventricle responds to this increased pressure load -y undergoing hypertrophy*
i.e.* the wall thic/ness of the left ventricle increases as the myocytes enlarge. A similar process occurs when there is
systemic hypertension. )he increased -lood pressure in the upper -ody (a-ove the coarctation) also produces
arterial hypertrophy* increasing the thic/ness of the vessel walls.
)he wall thic/ness of the a-dominal aorta (choice ")* femoral artery (choice A)* and other arteries -elow the
constriction may decrease in response to the lower than normal pressures.
Although one might predict a lower-than-normal -lood flow to the /idneys (choice $)* leg muscles (choice D)* and
other organs -elow the coarctation* if the -ody can compensate fully for the coarctation* -lood flow will -e normal in
these low-pressure areas of the -ody.
A !:-year-old man suffering from glomerulonephritis has a creatinine clearance of :C m=,min. <is medical
records indicate that his creatinine clearance was HCC m=,min a-out H year ago. Assuming that there has -een
no change in his diet* which of the following changes can -e e'pected in this patient compared to H year ago?
A. A 9-fold decrease in -lood urea nitrogen concentration
". A 9-fold decrease in creatinine e'cretion rate
. A 9-fold increase in creatinine e'cretion rate
D. A 9-fold increase in creatinine rea-sorption
$. A 9-fold increase in plasma creatinine concentration
$'planation(
)he correct answer is $. reatinine clearance is used clinically to estimate glomerular filtration rate (40R).
)herefore* the :C; decrease in creatinine clearance in this patient suggests that 40R has decreased -y :C;
over the past year. "ecause creatinine is freely filtered -ut not rea-sor-ed (choice D)* the filtration rate and
e'cretion rate of creatinine are e.ual during steady state conditions. When 40R decreases* the rate of
creatinine e'cretion will also decrease* causing the rate of creatinine e'cretion to fall -elow the rate of
creatinine production. )he result is an increase in plasma creatinine concentration. When plasma creatinine
levels have increased -y 9-fold* normal amounts of creatinine can then again -e e'creted (compare to choices
" and ) -ecause the e'cretion rate of creatinine is e.ual to the product of 40R and plasma creatinine
concentration. 7n summary* when 40R decreases* the plasma creatinine concentration continues to increase
until the rate of creatinine filtration (and e'cretion) -ecomes e.ual to the rate of creatinine production -y the
-ody.
)he -lood urea nitrogen concentration (choice A) increases when 40R is reduced.
A !9-year-old aucasian woman with no history of prior surgery complains of e'treme wea/ness and fatiga-ility
that has persisted for the past : months. She reports nausea and stomach cramps and has had trou-le /eeping
solid foods down. She is hypotensive while sitting* and her -lood pressure falls even more upon standing. She
notes increased frec/ling around her eyes* and on e'amination* her palmar creases appear dar/ened. Serum
potassium is K.: m$.,=. Which of the following is the most li/ely diagnosis?
A. Addison@s disease
". onn@s syndrome
. ushing@s syndrome
D. &elson@s syndrome
$. Secondary adrenal insufficiency
0. )ertiary adrenal insufficiency
$'planation(
)he correct answer is A. Addison@s disease usually occurs -ecause of autoimmune destruction of the adrenal
corte' (all three Aones are typically involved)* resulting in decreased secretion of cortisol* aldosterone* and
adrenal androgens. <yperpigmentation is the classical physical finding* resulting from increased serum A)<
due to loss of negative feed-ac/ inhi-ition -y cortisol at the pituitary and,or hypothalamus. )he increase in
pigmentation may occur -ecause the first H> amino acids of A)< are identical to alpha-melanocyte stimulating
hormone. =ow serum levels of cortisol produce gastrointestinal symptoms such as nausea* vomiting* and
anore'ia. 0atiga-ility and wea/ness are almost always reported. "lood pressure is usually low and orthostatic
hypotension may -e present* -ecause arterioles are less responsive to the constrictor effects of
catecholamines in the a-sence of cortisol. )he cardiovascular symptoms are worsened -y the loss of -lood
volume due to aldosterone deficiency. <yper/alemia is a manifestation of the low serum aldosterone3
hyponatremia may also -e present.
onn@s syndrome (choice ") results from hypersecretion of aldosterone. 7t is characteriAed -y hypertension*
hypernatremia* and hypo/alemia.
ushing@s syndrome (choice ) occurs -ecause of e'cessive secretion of cortisol. 7t is characteriAed -y central
o-esity* -uffalo hump* moon facies* hypertension* and if anything* hypo/alemia.
&elson@s syndrome (choice D) results in e'treme hyperpigmentation. 7t can occur in patients who have had
adrenalectomy to treat ushing@s disease. 7t results -ecause of e'cessive secretion of A)< from a pituitary
adenoma that is no longer -eing restrained -y the suppressive effect of cortisol. )he patient mentioned a-ove
has no history of adrenalectomy.
"oth secondary (choice $) and tertiary (choice 0) adrenal insufficiency result in low serum levels of A)<. )he
su-se.uent hypocortisolism can produce the gastrointestinal complaints and fatiga-ility* -ut not hyper/alemia.
With deficiency of R< or A)<* serum aldosterone usually remains in the normal range* and signs of
mineralocorticoid deficiency are not present. 0urthermore* low serum levels of A)< would not produce
hyperpigmentation.
A K!-year-old woman had a cere-rovascular accident K months ago. <er past medical history is remar/a-le for
hyperthyroidism and atrial fi-rillation. She initially presented with slurred speech and right hemiparesis. )he
hemiparesis resolved* -ut her speech is still agrammatic and nonfluent* and she has difficulty finding words and
completing sentences. <er comprehension is intact* and she appears frustrated when she attempts to spea/.
)he remainder of the neurologic e'amination is normal. Which of the following -est descri-es her deficit?
A. Apra'ia
". "roca aphasia
. Dysarthria
D. 4lo-al aphasia
$. Wernic/e aphasia
$'planation(
)he correct answer is ". "roca aphasia* caused in this case -y an em-olus to the "roca area (inferior frontal
gyrus)* is often associated with hemiparesis. )he aphasia is characteriAed -y slow* nonfluent speech with
deficits in word finding. "ecause comprehension is normal* patients are typically aware of the pro-lem and
appear frustrated.
Apra'ia (choice A) is a deficit of purposeful movement caused -y a central lesion.
Dysarthria (choice ) is a deficit in speech articulation with normal grammar and word finding.
4lo-al aphasia (choice D) is usually caused -y large infarcts in the distri-ution of the middle cere-ral artery. 7t
is characteriAed -y elements of "roca and Wernic/e aphasia (i.e.* an ina-ility to generate or comprehend
fluent speech).
Wernic/e aphasia (choice $)* caused -y lesions in Wernic/e area* is characteriAed -y fluent speech that has a
normal tempo* -ut is filled with incorrect words and neologisms. )he patient is una-le to comprehend speech.
A !!-year-old woman presents with a chief complaint of a sharp* sta--ing pain in her chest for the past H9 hours.
A careful history reveals that the patient e'perienced a myocardial infarct > years ago. )he woman refuses to lie
down in the e'amining room and instead leans forward* stating that it allows her to -reathe more easily. 6hysical
e'amination is unremar/a-le. )he $4 demonstrates diffuse S) segment elevations with upright ) waves. hest
radiographs appear normal. reatine /inase (%" fraction) is normal. Which of the following is the most li/ely
diagnosis?
A. Acute pericarditis
". Dissecting aortic aneurysm
. %yocardial infarction
D. Sta-le angina
$. Dnsta-le angina
$'planation(
)he correct answer is A. All of the answer choices represent common cardiovascular causes of chest pain.
<owever* only pericarditis and dissecting aortic aneurysms will produce sharp* /nife-li/e pains. 6atients with
pericarditis relieve their pain -y sitting and leaning forward. )he characteristic $4 patterns of pericarditis
include diffuse S) elevations with upright ) waves. While a pericardial ru- is diagnostic of pericarditis* its
presence is not necessary for diagnosis* and the physical e'amination may well -e unrevealing. )ypically* in
uncomplicated pericarditis* -oth chest radiographs and cardiac isoenAyme levels appear normal.
6ericarditis can -e differentiated from dissecting aortic aneurysms (choice ") on the -asis of clinical findings.
)he pain associated with dissecting aortic aneurysms is usually unrelated to -reathing* while the pain
associated with pericarditis is related to -reathing.
%yocardial infarcts (choice )* as well as sta-le (choice D) and unsta-le angina (choice $)* typically produce
more visceral types of pain.
A >C-year old male complains of fatigue and diffuse s/eletal pain. <e has a history of rohn@s disease* which led
to resection of the terminal ileum > years previously. Steatorrhea and diarrhea have continued since the surgery.
=a-oratory tests show that serum calcium is I.: mg,d=* serum phosphate is 9.: mg,d=* and serum parathyroid
hormone (&-terminal) is I:C pg,m=. Which of the following is the most li/ely cause of these findings?
A. Esteoporosis
". 6aget@s disease
. 6rimary hypoparathyroidism
D. Renal failure
$. ?itamin D deficiency
$'planation(
)he correct answer is $. Whenever serum calcium and phosphate are -oth decreased* vitamin D deficiency
should -e considered. 7n this case* the vitamin D deficiency is due to fat mala-sorption* including the fat-solu-le
vitamin D* su-se.uent to ileal resection. 7f more than HCC cm of the ileum are removed* primary -ile acid
production -y the liver cannot /eep up with -ile salt loss in the stool. )he total -ile salt pool decreases and fat
a-sorption* including the fat-solu-le vitamins is poor. Serum calcium is low -ecause of decreased dietary
a-sorption. Serum parathyroid hormone increases in response to the low calcium. Serum phosphate is low
-ecause of decreased dietary a-sorption and increased renal e'cretion (due to the increased parathyroid
hormone). With vitamin D deficiency* the -ones demineraliAe* producing osteomalacia. linical manifestations
often go unnoticed. ?ague complaints of wea/ness and -one pain may -e present. Radiographs of -ones in
osteomalacia typically reveal the presence of pseudofractures along the inner aspects of the femur* the pu-ic
rami* and the outer edges of the scapulas* upper fi-ula* and metatarsals. )hese radiolucent -ands* which are
perpendicular to the -one surface* may occur -ecause of pulsations of ma+or arteries that cross the -one.
Esteoporosis (choice A) is characteriAed -y loss of -one mass* -oth matri' and mineral. 7t is usually
asymptomatic* and serum levels of calcium* phosphate* and parathyroid hormone are within the normal range.
)he first hint of -one loss comes -ecause of a fracture in the wrist* hip* or verte-ra. Dual-energy radiography*
or other similar techni.ues* can -e used to directly .uantify the degree of -one loss.
7n 6aget@s disease (choice ")* -one mineral turnover is increased compared to normal. 7ts cause is un/nown*
-ut may -e due to a slow virus that infects osteoclast cells. "oth osteo-last and osteoclast activity is increased
in focal areas of the -one. )he disease is usually asymptomatic* the chief complaint -eing -one pain over the
lesions. =a-oratory findings include increased serum al/aline phosphatase* -ut serum calcium and parathyroid
hormone levels are usually normal.
With primary hypoparathyroidism (choice )* serum calcium is decreased and serum parathyroid hormone is
decreased* -ut serum phosphate is also increased (not decreased). Serum phosphate is increased -ecause
urinary e'cretion is diminished su-se.uent to the decrease in parathyroid hormone.
7n renal failure (choice D)* an increase (not decrease) in serum phosphate su-se.uent to decreased urinary
e'cretion is a primary manifestation. Serum calcium is decreased -ecause the hyperphosphatemia drives the
e.uili-rium -etween calcium and phosphate toward hydro'yapatite crystals. )his* in turn* produces an increase
in parathyroid hormone secretion with su-se.uent -one demineraliAation (renal osteodystrophy).
A three-year-old child is -rought to the emergency room after inhaling a peanut. )he peanut has lodged in the
right mainstem -ronchus* largely occluding it. )he child is cyanotic* and non-invasive transcutaneous monitoring
reveals a 6E9 of KC mm <g. Which of the following mechanisms -est accounts for the child@s hypo'emia?
A. Decreased capacity of pulmonary diffusion
". Decreased 6E9 in inspired air
. <ypoventilation of central origin
D. <ypoventilation of peripheral origin
$. 7ne.ualities of ventilation and perfusion
$'planation(
)he correct answer is $. 7ne.ualities of ventilation and perfusion contri-ute to hypo'ia in many settings. 7n this
case* -lood goes to -oth lungs (perfusion)* -ut air is prevented from entering one of the lungs (ventilation).
"ecause the right lung is -eing perfused* -ut not ventilated* hypo'emia ensues when the deo'ygenated -lood
from the right lung mi'es with o'ygenated -lood from the left lung. 7f the inade.uate ventilation of the lung
persists long enough* the lung tissue itself can -e damaged* causing a secondary local dilation of arterioles*
ma/ing the pro-lem even worse. 6eanuts are notorious for producing this type of pro-lem in young children
-ecause of their siAe and shape* which allows them to lodge in the trachea or main -ronchus after aspiration.
Decreased diffusion capacity (choice A) can occur when the -lood-gas -arrier is thic/ened (e.g.* diffuse
interstitial fi-rosis* sarcoidosis* as-estosis* respiratory distress syndrome)* when the surface area of the
-lood-gas -arrier is reduced (e.g.* pneumonectomy* emphysema)* or when less hemoglo-in is availa-le to
carry o'ygen (e.g.* anemia* pulmonary em-olism).
Decreased 6E9 in inspired air (choice ") is seen at high altitudes and when the settings are wrong during
artificial ventilation.
<ypoventilation of central origin (choice ) is seen in morphine and -ar-iturate overdose.
<ypoventilation of peripheral origin (choice D) is seen in poliomyelitis and chest trauma.

Anda mungkin juga menyukai